Está en la página 1de 31

Test

1. vuelta Distancia

Pediatra

Test

1. vuelta Distancia

Pediatra
1.

Seale la armacin correcta respecto al test de Apgar:

2)

1)

3)

2)
3)
4)
5)

2.

5.

1
2
3
4
5

6.

2)
3)

4)
5)

La alimentacin debe iniciarse siempre por sonda nasogstrica


en los menores de 32 semanas de edad gestacional, por la
falta de coordinacin entre succin, deglucin y respiracin.
En los casos que necesiten tratamiento con eritropoyetina,
debe iniciarse a la vez tratamiento con hierro va oral.
Las necesidades de lquidos son mayores que en los neonatos
a trmino, tanto ms cuanto ms prematuro sea el recin
nacido.
Debe aumentarse el intervalo de dosis de todos los medicamentos de eliminacin renal.
Hay que realizar a todos ecografa transfontanelar aunque
no tengan alteraciones en la exploracin neurolgica.

Seale la armacin FALSA respecto a las lesiones producidas


por traumatismos obsttricos:
1)

El cefalohematoma es una hemorragia subperistica, y est


delimitado por las suturas craneales.

CTO Medicina C/Francisco Silvela, 106 28002 - Madrid

Cul de los siguientes fenmenos dermatolgicos que podemos


encontrar en un neonato NO debe sugerirnos una patologa
subyacente?
1)
2)
3)
4)
5)

Cul de las siguientes armaciones sobre los cuidados del


recin nacido prematuro es FALSA?
1)

4.

4)
5)

RN que, al minuto de vida, presenta FC de 110 latidos por


minuto, acrocianosis con esfuerzo respiratorio ausente, as como
hipotona y leve mueca al introducir la sonda de aspiracin. La
puntuacin de Apgar ser:
1)
2)
3)
4)
5)

3.

Se debe practicar nicamente a los RN patolgicos o supuestamente patolgicos.


Entre los parmetros que se valoran se encuentra la frecuencia
respiratoria.
Se debe realizar a los 1-5-10 minutos de vida.
Un test de Apgar de 4, al minuto de vida, implica un alto
riesgo de parlisis cerebral.
La palidez cutnea generalizada supone una puntuacin
de 1 en el test de Apgar.

Manchas hipopigmentadas geogrcas.


Ictericia precoz en las primeras 24 horas.
Nevus ammeus.
Melanosis pustulosa.
Manchas caf con leche.

Ante un neonato a trmino que tras cesrea presenta precozmente taquipnea y cianosis que desaparece con pequeas
cantidades de oxgeno, en el que la auscultacin pulmonar es
normal y en el que la radiografa muestra marcas vasculares
prominentes y lquido en las cisuras sin broncograma areo,
sospecharemos:
1)
2)
3)
4)
5)

7.

La clavcula es el hueso que se fractura con mayor frecuencia


durante el parto.
El llamado ndulo de Stroemayer suele requerir tratamiento quirrgico.
La rotura heptica no siempre precisa tratamiento quirrgico.
Las fracturas craneales ms frecuentes en el parto son las
lineales.

Enfermedad de membrana hialina (EMH).


Taquipnea transitoria del recin nacido.
Sndrome de aspiracin meconial (SAM).
Persistencia de la circulacin fetal (PCF).
Displasia broncopulmonar (DBP).

RN de 35 semanas de edad gestacional que en las primeras 12


horas de vida presenta taquipnea, quejido audible y cianosis
progresiva, refractaria a la administracin de oxgeno. A la
auscultacin encontramos crepitantes, sobre todo en bases
pulmonares. En la gasometra aparece hipoxemia progresiva,
aumento de la PCO2 y acidosis metablica. Qu radiografa de
trax esperara encontrar en este momento?
1)
2)
3)
4)

Normal.
Inltrado de patrn reticulogranular con broncograma areo.
Hiperinsuacin y aumento de las marcas vasculares, sin
broncograma.
Patrn de esponja.

Tfno. (0034) 91 782 43 30/33/34 E-mail: secretaria@ctomedicina.com www. ctomedicina.com

Test

1. vuelta Distancia

Pediatra
5)
8.

En el tratamiento de la EMH se incluyen las siguientes medidas,


EXCEPTO:
1)
2)
3)
4)
5)

9.

5)

Borde de ambos pulmones colapsados.

Administracin de surfactante endotraqueal.


Antibioterapia.
Administracin de oxgeno y ventilacin mecnica, si es
preciso.
Prostaglandinas.
Control del aporte de lquidos.

12.

Un nio de 42 semanas de gestacin, arrugado, plido, hipotnico, apneico y cubierto de lquido amnitico verdoso. Respecto
al cuadro que puede padecer este paciente, seale la opcin
FALSA:

Recin nacido a trmino que presenta dicultad respiratoria


inmediata, con importante tiraje intercostal y supraesternal,
taquipnea, trax hiperinsuado y abdomen excavado. En la
auscultacin pulmonar destaca abolicin casi total del murmullo vesicular en el hemitrax izquierdo, con tonos cardacos
desplazados a la derecha. Seale la respuesta verdadera acerca
de la patologa que sospecha:
1)
2)

1)
2)
3)
4)
5)

La primera maniobra en su reanimacin debe ser la aspiracin


de trquea bajo visin laringoscpica.
Es un patologa tpica de RNPT y RNT.
E. coli es el germen ms frecuente de infeccin bacteriana
en estos pacientes.
En la Rx de trax es tpico encontrar un patrn de atrapamiento areo.
Esta patologa produce una llamativa hipertensin pulmonar.

3)
4)
5)

13.
10.

Recin nacido de 38 semanas que tras expulsivo dicultoso


presenta el brazo izquierdo pegado al tronco junto con cierta
rotacin interna de dicho miembro. Seale la respuesta verdadera respecto a la lesin obsttrica que sospecha en este recin
nacido:
1)
2)
3)
4)
5)

11.

El reejo de Moro no estar presente en el lado izquierdo.


Se debe a una lesin en las races inferiores del plexo braquial.
Este tipo de parlisis es menos frecuente que la parlisis de
Klumpke.
Es caracterstica la presencia de mano cada.
Suele asociarse con sndrome de Horner.

Un RN de 32 semanas, con antecedente de enfermedad de


membrana hialina sometido a ventilacin mecnica con altas
concentraciones de oxgeno, tiene actualmente un mes de vida
y sigue dependiendo del respirador para mantener adecuadas
saturaciones de oxgeno. En los ltimos das presenta adems
oliguria con edemas en miembros inferiores y se palpa el
hgado a 5 cm bajo el reborde costal derecho. En la Rx de trax
se observa una imagen de pequeas zonas redondas y claras
que alternan con otras de mayor densidad. En relacin con
la patologa que presenta este paciente, seale la respuesta
FALSA:

2)
3)
4)

Se consideran factores de riesgo para su desarrollo tener


distrs respiratorio severo que precise largos perodos de
administracin de O2.
Las principales causas de muerte de estos pacientes son la
bronquiolitis necrotizante y fallo cardaco derecho.
El tratamiento incluye broncodilatadores, diurticos y dexametasona.
La nefrolitiasis puede ser una complicacin de estos
pacientes, secundaria al uso de diurticos y a la alimentacin
parenteral.

1)
2)

CTO Medicina C/Francisco Silvela, 106 28002 - Madrid

4)
5)

15.

Puncin lumbar y anlisis del LCR.


Ecocardiograma.
Ecografa cerebral.
TC craneal.
Rx trax.

Un RN pretrmino de 1.800 g de peso, con antecedentes de


hipoxia perinatal, a los 10 das de vida presenta distensin
abdominal importante, vmitos y deposiciones hemorrgicas.
En la Rx de abdomen se observa edema de asas intestinales,
con un patrn en miga de pan y presencia de gas en la pared
intestinal. Cul sera la opcin teraputica ms adecuada en
este paciente?

3)
1)

En ms del 50% de los casos hay otras malformaciones


asociadas.
El tratamiento es quirrgico, y debe realizarse lo antes posible,
tratando despus las alteraciones que haya asociadas.
El diagnstico diferencial con malformaciones pulmonares
qusticas se hace con ecografa.
El reujo gastroesofgico es una secuela rara tras la ciruga.
Los afectados suelen desarrollar hipertensin pulmonar a
partir del nacimiento, que en los casos graves condiciona
su pronstico.

RNPT de 29 semanas de edad gestacional sufre distrs respiratorio desde el nacimiento. A las 36 horas de vida presenta
hipotensin, bradicardia, cianosis y la fontanela se encuentra
abombada. En el hemograma presenta 15.000 leucocitos, neutrlos 45%, linfocitos 42%, cayados 0%, plaquetas 180.000,
hematocrito 30%. Qu prueba complementaria considerara
ms adecuada en este momento?
1)
2)
3)
4)
5)

14.

A pesar de que pueda retirarse la administracin de O2 antes


de salir de la unidad de cuidados intensivos, el pronstico
para la mayora de estos pacientes es malo, presentando
en la adolescencia obstruccin de las vas respiratorias e
hiperreactividad bronquial.

Enema de solucin hiperosmolar.


Alimentacin enteral a dbito continuo a travs de una
sonda nasogstrica.
Actitud expectante y vigilancia cuidadosa, ante la posibilidad
de perforacin intestinal.
Se debe suspender la alimentacin enteral y pautar uidoterapia y antibioterapia i.v.
La presencia de neumatosis intestinal aconseja la intervencin quirrgica urgente.

RN de 38 semanas de edad gestacional presenta distensin


abdominal y vmitos biliosos durante el primer da de vida. En
el examen fsico se palpan cordones duros que siguen el marco
clico. Una radiografa en bipedestacin muestra una masa en

Tfno. (0034) 91 782 43 30/33/34 E-mail: secretaria@ctomedicina.com www. ctomedicina.com

Test

1. vuelta Distancia

Pediatra
pompa de jabn en cuadrante inferior derecho. El tratamiento
inmediato ms apropiado es:
1)
2)
3)
4)
5)
16.

20.

Respecto a la anemia en el perodo neonatal, seale cul de las


siguientes armaciones es FALSA:
1)

Antibioterapia.
Supositorios de glicerina.
Enema de solucin hiperosmolar.
Estimulacin rectal.
Ciruga.

2)
3)
4)

Seale la armacin FALSA, de entre las siguientes, respecto a


la ictericia siolgica:

5)
1)
2)
3)
4)
5)
17.

Seale la armacin FALSA, de entre las siguientes, respecto a


la isoinmunizacin Rh:
1)
2)
3)

1)

4)

3)
4)

5)

Su frecuencia es aproximadamente de 1 por cada 200 RN


que reciben lactancia materna.
Es una ictericia que aparece a la mitad o nal de la primera
semana, alcanzando su mximo en la tercera.
Se pueden alcanzar cifras de hasta 30 mg/dl, de ah el riesgo
de kernicterus en estos neonatos.
En la patogenia se implican varias sustancias presentes en la
leche materna como pregnanodiol, cidos grasos de cadena
larga y glucuronidasas.
Cuando se suspende la lactancia materna durante 2-4
das se produce una rpida disminucin de la bilirrubina
srica.

5)

22.

La madre suele ser 0.


Los anticuerpos causantes suelen ser de la clase IgG.
Pueden resultar afectos los productos del primer embarazo.
El antgeno ms frecuentemente implicado es A1.
La enfermedad se agrava en los embarazos siguientes, si no
hacemos prolaxis.

2)
3)
4)

5)

23.

24.
1)
2)
3)
4)
5)

Se debe a un dcit de los factores dependientes de


vitamina K.
Suele iniciarse a partir de las primeras 24 horas.
Es ms frecuente en hijos de madres que han tomado fenitona o fenobarbital.
Las complicaciones hemorrgicas son menos frecuentes en
nios que toman LM que en los que reciben bibern.
Se trata con la administracin de vitamina K y/o plasma
fresco congelado.

CTO Medicina C/Francisco Silvela, 106 28002 - Madrid

Megalencefalia.
Sndrome asfctico.
Hiperbilirrubinemia.
Sndrome de regresin caudal.
Cardiomiopata.

En la sepsis del RN, los grmenes ms frecuentes son:


1)
2)
3)
4)
5)

25.

Se observa en el 75% de los hijos de madres diabticas y en


el 25% de los hijos de madres con diabetes gestacional.
Aparece en las primeras 24 horas de vida y suele ser asintomtica.
Se piensa que es debida a hiperinsulinismo fetal.
La probabilidad de que el RN presente hipoglucemia es ms
elevada cuanto ms altas se encuentren las cifras de glucosa
en sangre de cordn.
Se trata mediante infusin i.v. de bolos de glucosa y debe
retrasarse la alimentacin oral para minimizar el riesgo de
hipoglucemia de rebote.

NO es propio del RN hijo de madre diabtica:


1)
2)
3)
4)
5)

Un lactante de una semana es llevado a Urgencias en estado de


coma con hemorragias en la retina y palidez importante. Entre
sus antecedentes destaca que el parto tuvo lugar en el domicilio, no present complicaciones posteriores y est recibiendo
lactancia materna. Seale lo FALSO respecto a este cuadro:

La enfermedad hemoltica grave suele afectar al segundo hijo.


Suele cursar con ictericia, que aparece en las primeras 24
horas de vida.
Debe hacerse prolaxis con gammaglobulina especca
anti-D antes de que pasen 72 horas del parto, cuando el
test de Coombs directo es positivo.
Cuando las cifras de bilirrubina superan los 20 mg/dl, es
necesario realizar una exanguinotransfusin.
El 90% de los casos de incompatibilidad se deben al antgeno D.

Respecto a la hipoglucemia del hijo de madre diabtica, seale


cul de las siguientes armaciones es FALSA:
1)

En relacin a la enfermedad hemoltica por isosensibilizacin


AB0, seale la FALSA:
1)
2)
3)
4)
5)

19.

21.

En relacin a la ictericia por lactancia materna o sndrome de


Arias es FALSO que:

2)

18.

Las cifras mximas de bilirrubina suelen alcanzarse a los 3-4


das.
La ictericia en los RNPT suele ser de inicio algo ms tardo y
ms prolongada que en los RNT.
Puede comenzar en las primeras 24 horas de vida en el RNT.
Su duracin suele ser inferior a 10-15 das.
En los RNPT pueden alcanzarse cifras de hasta 14 mg/dl.

La anemia siolgica es ms intensa y ms precoz en los


RNT que en los RNPT.
El cuadro de anemia siolgica neonatal est en relacin con la
hemlisis de glbulos rojos y con un dcit de eritropoyetina.
En los RNPT est indicada la administracin de suplementos
de hierro a partir de los 2 meses de vida.
La anemia siolgica puede precisar transfusin en algunos
casos.
La betatalasemia no se manifestar antes de los 6 meses de
vida.

Klebsiella.
E. coli y estreptococos del grupo A.
E. coli y estreptococos del grupo B.
Estalococo.
Gardnerella vaginalis.

Qu hara usted ante un RN de madre con hepatitis B activa en


el momento del nacimiento?

Tfno. (0034) 91 782 43 30/33/34 E-mail: secretaria@ctomedicina.com www. ctomedicina.com

Test

1. vuelta Distancia

Pediatra
1)
2)
3)
4)
5)
26.

Observar la aparicin de los sntomas y luego tratar.


Extraer sangre de cordn para estudiar transaminasas.
Exanguinotransfusin inmediata, si la madre tiene el antgeno HBe positivo.
Poner una dosis bimensual de gammaglobulina.
Iniciar vacunacin y gammaglobulina en paritorio.

30.

1)
2)
3)
4)
5)

Un RN de bajo peso para su edad gestacional presenta un


exantema petequial, hepatoesplenomegalia y coriorretinitis. En
la TC craneal se observan calcicaciones periventriculares sin
otros hallazgos. Con respecto al cuadro que parece presentar
este paciente, seale la verdadera:
31.
1)
2)
3)
4)
5)

27.

Es la segunda causa ms frecuente de infeccin viral congnita.


El 90% de los recin nacidos infectados estarn asintomticos al nacer.
La manifestacin ms tarda y caracterstica es una hipoacusia
de transmisin.
La afectacin ocular tpica es una retinitis pigmentosa.
Estos nios suelen desarrollar macrocefalia evolutivamente.

32.

28.

2)

3)
4)
5)

Cuando la madre contrae la infeccin en el tercer trimestre


aumenta el riesgo de infeccin del feto, pero el cuadro clnico
es ms leve.
Si la infeccin se produce despus de la 12. semana de
gestacin, es poco probable la asociacin de una cardiopata
congnita.
La presencia de IgM especca antirrubola reeja la produccin en el tero de anticuerpos por el feto.
El lactante puede permanecer con infeccin crnica durante
meses despus del nacimiento.
El virus puede aislarse en sangre.

De los siguientes, el factor ms relacionado con el crecimiento


intrauterino retardado tipo simtrico es la:

3)
4)
5)

33.

Infeccin congnita.
Isoinmunizacin Rh.
Hipertensin.
Diabetes con vasculopata.
Edad de la mujer por encima de 35 aos.

CTO Medicina C/Francisco Silvela, 106 28002 - Madrid

La sonrisa social referencial aparece ente el 1. y 2. mes.


El sostn ceflico se inicia a los 3 meses.
La sedestacin estable se consigue a los 6-8 meses.
Emite sus primeros fonemas con 3-4 semanas.
Realiza frases cortas con 2 aos.

En relacin con el crecimiento y el desarrollo del nio en su


primer ao de vida, cul de las siguientes armaciones es
FALSA?
1)
2)
3)

1)
2)
3)
4)
5)

Su causa ms frecuente es la disgenesia tiroidea.


Situaciones de dismorfognesis pueden asociarse a sordera
neurosensorial.
Un tratamiento precoz supone un mejor pronstico neurolgico.
Los signos clnicos del hipotiroidismo estn ya presentes
inmediatamente al nacimiento.
En situaciones de mal control pueden acabar desencadenando una pubertad precoz.

En relacin con el desarrollo psicomotor en el nio, seale la


respuesta FALSA:
1)
2)
3)
4)
5)

34.

Onfalocele.
Granuloma umbilical.
Quiste o persistencia del uraco.
Gastrosquisis.
Ombligo amnitico.

Recin nacida, de 3 semanas de vida, que presenta valores de TSH


de 75 g/ml (0,3 5 ) en el cribado neonatal de hipotiroidismo.
Con respecto a esta situacin, es FALSO:
1)
2)

Un RN de 38 semanas de edad gestacional y 1.750 g, presenta


erupcin purpuropetequial, hepatoesplenomegalia y se
ausculta un soplo continuo spero en 2. espacio intercostal
izquierdo. En la Rx de trax se objetiva cardiomegalia con
aumento de la vascularizacin pulmonar as como imgenes
lineales radiotransparentes con aumento de densidad sea
en la zona metasaria del hmero sin reaccin peristica. En
relacin con el cuadro que presenta este paciente, seale la
respuesta FALSA:
1)

29.

Coriorretinitis.
Cataratas.
Crisis convulsivas.
Hidrocefalia.
Calcicaciones cerebrales.

Infeccin por VIH.


Infeccin por CMV.
Infeccin por VHS tipo I.
Infeccin por VHS tipo II.
Exantema txico del RN.

Un recin nacido de 3 semanas de vida que desprendi el cordn


umbilical a los 16 das y cuyo ombligo no ha cicatrizado, presenta una pequea tumoracin sonrosada con secrecin seromucosa con un pH de 7,4. Cul de los siguientes diagnsticos
le parece ms probable?
1)
2)
3)
4)
5)

Cul de las siguientes manifestaciones NO forma parte de la


ttrada de Sabin?
1)
2)
3)
4)
5)

Un RN hijo de madre prostituta presenta en el 7. da de vida un


cuadro de ebre con letargia y rechazo de las tomas as como
convulsiones. A la exploracin destaca hepatoesplenomegalia
con fontanela anterior abombada y lesiones vesiculares en calota craneal. Cul sera la causa ms probable de este cuadro?

4)
5)

El nio aproximadamente duplica su peso al 5. mes y lo


triplica al ao de vida.
El peso del RN puede disminuir un 5-10% en la primera
semana de vida por prdida de lquido extravascular.
Los reejos precoces o arcaicos presentes al nacimiento
suelen desaparecer entre los 3 y 4 meses de edad.
Hacia los 2 meses suele haber hecho su aparicin la sonrisa
social.
Duplican la talla del nacimiento poco despus de cumplir
el primer ao de edad.

Tfno. (0034) 91 782 43 30/33/34 E-mail: secretaria@ctomedicina.com www. ctomedicina.com

Test

1. vuelta Distancia

Pediatra
35.

Con respecto al calostro, es FALSO que:


1)
2)
3)
4)
5)

36.

4)
5)

Se denomina as a la secrecin mamaria durante los primeros


2-4 das postparto.
Contiene ms protenas y ms carbohidratos que la leche
humana madura.
Su densidad es mayor que la de la leche humana madura.
Las caractersticas de leche humana madura se adquieren
alrededor de la 3.-4. semana.
Posee factores inmunolgicos.

40.

1)
2)
3)
4)
5)

La leche de vaca tiene un contenido energtico dos veces


superior al de la leche materna.
La leche de vaca contiene una cantidad de calcio superior
a la leche materna.
La leche materna contiene mayor cantidad de vitamina K
que la leche de vaca.
La leche de vaca es ms rica en hidratos de carbono que la
leche materna.
La leche materna tiene una relacin casena/seroprotenas
de 70/30.

Ante un nio de 2 aos de edad que, estando previamente bien,


se despierta bruscamente por la noche con tos perruna, estridor
y dicultad respiratoria, pulso acelerado y afebril, pensaremos
en:
1)
2)
3)
4)
5)

Respecto a las diferencias entre la leche materna y la leche de


vaca, seale la armacin correcta:
41.

La estatura de un nio de 4 aos est por debajo del percentil


3. Su velocidad de crecimiento es normal. La edad sea es de
2 aos. El padre present un retraso en la pubertad, aunque
actualmente su talla est dentro de los lmites normales. Este
proceso se denomina:

2)
3)
4)
5)

1)
2)
3)
4)
5)
38.

42.

Acude a su consulta un padre con su hijo de 8 aos preocupado


porque dice que su hijo es el nio ms bajo de toda su clase.
Su talla actual se encuentra en el percentil 2 y su talla gentica
est situada en el percentil 3. Al realizar el estudio de la edad
sea del nio se ve que sta coincide con su edad cronolgica.
Qu cree usted que explicara la talla baja de este nio?
1)
2)
3)
4)
5)

39.

Hipopituitarismo primario.
Hipopituitarismo secundario.
Retraso constitucional del crecimiento.
Talla baja gentica.
Sndrome de Laron.

Un hipocrecimiento por deciencia de la hormona de crecimiento.


Una alteracin cromosmica.
Un hipotiroidismo congnito.
Un retraso constitucional del crecimiento.
Una talla baja familiar.

Nia de 2 aos que acude a Urgencias por cuadro de disnea,


estridor y tos perruna. En la exploracin destaca rinorrea
intensa, amgdalas hipertrcas con exudado puntiforme.
Fiebre de 38,5 C. A la auscultacin pulmonar se aprecia hipoventilacin bilateral y estridor inspiratorio. El diagnstico ms
probable ser:
1)
2)
3)

Traquetis bacteriana.
Laringitis supragltica.
Laringitis aguda.

CTO Medicina C/Francisco Silvela, 106 28002 - Madrid

Aplicar oxgeno e intentar obtener una va area articial


en quirfano.
Iniciar tratamiento antibitico con ceftriaxona.
Aplicar adrenalina racmica y corticoides.
Usar las medidas generales para descender la hipertermia
que sufre el paciente.
Mantener tranquilo al paciente.

Lactante varn de 3 meses que desde hace dos das presenta


cuadro de rinorrea serosa con estornudos y tos seca acompaado
de rechazo de las tomas. Hoy acude a Urgencias por cuadro de
dicultad respiratoria con taquipnea y tos sibilante paroxstica.
Exploracin fsica: 37,7 C, regular estado general, aciantico,
con aleteo nasal y tiraje subcostal e intercostal. Auscultacin pulmonar: sibilancias espiratorias difusas con espiracin alargada.
Se realiza hemograma con 3,5 millones de hemates; Hb de 10
g/dl; Hto de 32%; 7.000 leucocitos/mm3 con 50% neutrlos,
35% linfocitos y 5% monocitos. En la Rx de trax destaca hiperinsuacin pulmonar. Seale la FALSA respecto a la evolucin
clnica de esta enfermedad:
1)
2)
3)
4)
5)

43.

Laringotraquetis aguda.
Laringitis espasmdica.
Epiglotitis aguda.
Difteria.
Traquetis bacteriana.

Varn de 5 aos que, estando previamente bien, despierta por la


noche con ebre de 39,8 C, voz apagada, intenso babeo, estridor
inspiratorio con escasa tos. A la exploracin destaca mal estado
general, estridor inspiratorio e intenso tiraje supra-esternal y
subcostal, as como tendencia a la hiperextensin del cuello. Se
realiza radiografa lateral de faringe, observando ocupacin de
espacio supragltico. En el hemograma destaca una leucocitosis
con desviacin izquierda. Qu medida teraputica NO estara
aconsejada en este caso?
1)

37.

Laringitis estridulosa.
Epiglotitis.

La fase ms crtica son las primeras 48-72 horas desde el


comienzo de la tos y la disnea.
En menores de un mes puede cursar como apnea con escasez
de clnica respiratoria.
Despus de la fase crtica mejoran muy rpidamente.
La ribavirina podra usarse en lactantes con cardiopatas
congnitas o displasia broncopulmonar.
Las complicaciones bacterianas como la bronconeumona
u OMA son frecuentes.

Paciente de 2 aos de edad consulta por estancamiento ponderoestatural (< P3). Entre los antecedentes personales destacan
en el ltimo ao episodios repetidos de bronquiolitis con regular
recuperacin posterior, persistiendo una tos crnica. Adems
presenta diarrea intermitente sin acompaarse de ebre ni
vmitos. Cul sera el diagnstico ms probable?

Tfno. (0034) 91 782 43 30/33/34 E-mail: secretaria@ctomedicina.com www. ctomedicina.com

Test

1. vuelta Distancia

Pediatra
1)
2)
3)
4)
5)

Enfermedad celaca.
Tuberculosis.
Asma.
Fibrosis qustica.
Dcit de alfa-1 antitripsina.

1)
2)
3)
4)
5)

44.

Sobre la traquetis bacteriana, todo es cierto EXCEPTO:


49.
1)
2)
3)
4)
5)

45.

2)
3)
4)

5)

3)
4)
5)

RN de pocas horas de vida que presenta exceso de secreciones


orales, episodios de cianosis y atragantamiento durante las
tomas junto con un abdomen muy distendido y timpnico a
la percusin. Entre sus antecedentes destacan polihidramnios
materno durante la gestacin e imposibilidad de pasarle una
sonda nasogstrica hasta el estmago. En relacin con la patologa que sospecha, reera cul de las complicaciones siguientes
al tratamiento quirrgico es la ms frecuente:
1)
2)
3)
4)
5)

50.

La sioterapia respiratoria alarga la supervivencia.


Los productos enzimticos micronizados disminuyen la
esteatorrea.
Se necesita aportar suplementos de vitaminas liposolubles
a la dieta.
La colonizacin de la va respiratoria por P. aeruginosa es
fcilmente revertida con la antibioterapia adecuada.
En ocasiones son necesarios lavados y aspiracin traqueobronquial broncoscpica.

51.

El agente causal ms frecuente en la infancia es el rotavirus.


Suele ser un proceso autolimitado, que cede en 3-10 das.
Puede causar deshidratacin isotnica con acidosis.
Es frecuente que tienda a la cronicacin.
El diagnstico puede hacerse mediante deteccin rpida
de antgeno de rotavirus en heces.

Lactante de 3 meses, alimentado al pecho hasta hace una semana


en que, debido a una hipogalactia, se decide suplementar con
una frmula articial. Hoy acude a Urgencias por presentar
anorexia, vmitos, prdida de peso as como exantema urticarial en tronco. Entre sus antecedentes destaca: RNT, nacido
por cesrea y haber iniciado alimentacin oral a las 4 horas de
vida con frmula articial. Con respecto a la patologa actual,
es FALSO que:

CTO Medicina C/Francisco Silvela, 106 28002 - Madrid

4)
5)

Suele ocurrir en la primera semana de vida.


Es ms frecuente en nias.
Presenta alcalosis metablica hiperclormica con hipopotasemia.
La prueba diagnstica de eleccin es la Rx de abdomen.
El tratamiento consiste en la pilorotoma submucosa de
Ramstedt.

RN con sndrome de Down presenta vmitos biliosos a las 36


horas de vida. A la exploracin destaca abdomen excavado. El
diagnstico ms probable es:
1)
2)
3)
4)
5)

53.

Cambiar su frmula por un hidrolizado de protenas de vaca.


Hacer endoscopia para descartar esofagitis.
Solicitar ecografa abdominal y/o trnsito digestivo.
Realizar manometra esofgica.
Indicar a la madre medidas posturales y usar espesante de
la leche.

Seale la respuesta verdadera respecto a la estenosis hipertrca


de ploro:
1)
2)
3)

52.

Fstula anastomtica.
Estenosis esofgica.
Fstula traqueoesofgica recidivante.
Reujo gastroesofgico.
Traqueomalacia.

Varn de 3 meses que consulta por vmitos desde hace un mes.


Vomita despus de todas las tomas, sin fuerza; come siempre
con buen apetito y presenta adecuado desarrollo ponderoestatural, sin prdida de peso. Realiza dos deposiciones diarias
con heces de consistencia, aspecto y volumen normal. Entre sus
antecedentes destaca estar alimentado con frmula articial
desde el mes de vida. Su actitud sera:
1)
2)
3)
4)
5)

Varn de 13 meses que desde hace 2 das presenta ebre de


38 C y sntomas catarrales. Hoy acude por presentar vmitos
no biliosos y 6 deposiciones en las ltimas 2 horas, lquidas, no
ftidas, sin moco ni pus ni sangre. Respecto al cuadro que presenta este paciente, seale cul de las siguientes armaciones
es FALSA:
1)
2)
3)
4)
5)

48.

La lesin anatomopatolgica pulmonar ms precoz es la


bronquiolitis.
Un 10% de los casos debuta con un leo meconial.
La funcin endocrina del pncreas se altera con el tiempo.
La aspergilosis broncopulmonar alrgica es excepcional en
estos pacientes y se trata con antifngicos como el voriconazol.
Pueden presentar deshidratacin con alcalosis hipoclormica
en ambientes calurosos.

En el tratamiento de la brosis qustica, NO es cierto que:


1)
2)

47.

La causa ms frecuente es Haemophilus inuenzae.


Debe sospecharse ante todo crup viral con ebre que no
responde al tratamiento.
No estarn presentes los hallazgos clnicos de una epiglotitis.
Ser necesario instaurar tratamiento antibitico.
En ocasiones puede requerir uso de va area articial.

Seale la respuesta FALSA, al hablar de la brosis qustica:


1)

46.

No suele haber antecedentes familiares de atopia.


La protena comnmente implicada es la betalactoglobulina.
Las pruebas cutneas sern positivas.
El diagnstico denitivo se realiza mediante un test de
supresin-provocacin.
Suele ser un proceso transitorio.

Estenosis hipertrca de ploro.


Enfermedad de Hirschprung.
Divertculo de Meckel.
Atresia duodenal.
Invaginacin intestinal.

Con respecto al divertculo de Meckel, es FALSO que:


1)

Afecta al 1-2% de la poblacin.

Tfno. (0034) 91 782 43 30/33/34 E-mail: secretaria@ctomedicina.com www. ctomedicina.com

Test

1. vuelta Distancia

Pediatra
2)
3)
4)
5)

54.

Un varn de 14 meses, previamente sano, de forma brusca


presenta crisis de llanto con encogimiento de piernas y palidez,
separados por perodos asintomticos de duracin variable en
los que permanece decado. Ha presentado dos vmitos y una
deposicin semilquida sanguinolenta. Exploracin fsica: afebril;
palpacin de una masa alargada y mal denida, localizada en
hipocondrio derecho. En la Rx de abdomen aparece silencio
areo en hipocondrio derecho. Seale cul de las siguientes
armaciones es cierta respecto al cuadro que presenta este
paciente:
1)
2)
3)
4)
5)

55.

Debe retrasarse el tratamiento durante 4-6 horas debido a


la alta incidencia de reduccin espontnea.
La recidiva es menos frecuente despus de la reduccin
hidrosttica que tras la reduccin quirrgica.
Si hay signos de obstruccin intestinal de 48 horas de evolucin, es preferible la reduccin quirrgica.
Una invaginacin asociada a divertculo de Meckel suele
reducirse satisfactoriamente con enema opaco.
La mortalidad es muy elevada, aunque la reduccin tenga
lugar en las primeras 24 horas.

57.

Biopsia rectal patolgica.


Heces acintadas.
Ampolla sin heces en el tacto rectal.
Relajacin del esfnter anal interno.
Permanencia del bario en colon durante ms de 24 horas al
realizar un trnsito digestivo.

Paciente de 12 meses que, de forma progresiva, inicia un cuadro


de inapetencia, mal carcter, retraso del crecimiento ponderoestatural, as como diarrea abundante, ftida y de aspecto
grasiento. Exploracin fsica: afebril, regular estado general,
palidez cutaneomucosa, prdida de masa muscular proximal y
abdomen distendido. Seale lo FALSO respecto a la enfermedad
celaca:
1)
2)
3)
4)
5)

Existe una predisposicin gentica, relacionada con los HLA


B8, DR7, DR3 y DQ2.
Hay una mayor prevalencia de la enfermedad en nios con
dcit selectivo de IgA o diabetes mellitus.
El perodo ms comn de presentacin es entre los 6 meses
y los 2 aos de edad.
Los anticuerpos antiendomisio IgA tienen una alta especicidad y sensibilidad.
El trigo, arroz, cebada y centeno deben ser excluidos de la
dieta para toda la vida.

CTO Medicina C/Francisco Silvela, 106 28002 - Madrid

Ante una enferma con clnica sospechosa de enfermedad celaca,


seale lo correcto respecto a su diagnstico:
1)
2)
3)
4)
5)

58.

5)
59.

Se puede asociar a poliesplenia.


El tratamiento denitivo es la hepatoportoenterostoma.
En la biopsia es posible hallar clulas gigantes.
Es infrecuente la aparicin de varios casos en una misma
familia.
En el sondaje duodenal se suele observar ausencia de bilis.

Paciente de 5 meses que acude por presentar, desde hace 2 das,


hasta 10 deposiciones al da, lquidas sin moco ni sangre, as
como dolor medio abdominal sin relacin temporal. Entre sus
antecedentes destaca diarrea sanguinolenta hace una semana.
En la exploracin destaca: afebril, buen estado general, distensin abdominal con abundantes ruidos hidroareos y eritema
en regin del paal. En el examen macroscpico de heces se
detecta presencia de azcares reductores. Este cuadro es:
1)
2)
3)
4)
5)

60.

Ante la sospecha muy alta con clnica orida, se debe excluir


el gluten de la dieta, sin necesidad de realizar biopsia.
Los Acs antitransglutaminasa son los ms especcos.
La anatoma patolgica de la muestra de biopsia es patognomnica.
Antes de realizar la primera biopsia, es necesario excluir el
gluten.
El control de la enfermedad se basa en la realizacin de
biopsias anuales.

Neonato varn, de 3 semanas de vida, acude a su consulta por


ictericia con coluria e hipocolia. A la exploracin se palpa hepatomegalia. En la gammagraa con HIDA, la captacin es normal,
pero la excrecin es nula. Sobre la entidad que sospecha, seale
la opcin FALSA:
1)
2)
3)
4)

Nio de 4 meses que es llevado a consulta por estreimiento


crnico desde el nacimiento. Entre sus antecedentes personales
destaca tapn meconial al nacimiento que se resolvi con
enemas. Alimentacin materna exclusiva. Talla y peso < P3.
Cul de los siguientes datos NO apoyara el diagnstico?
1)
2)
3)
4)
5)

56.

Se localiza en leon a unos 75 cm de la vlvula ileocecal.


Se denomina hernia de Littr cuando el divertculo de Meckel
se aloja en una hernia inguinal indirecta.
Su manifestacin clnica ms frecuente es una hemorragia
gastrointestinal acompaada de dolor abdominal.
El mtodo diagnstico ms til es la gammagrafa con
Tc-99.

Dcit de sacarasa-isomaltasa.
Dcit aislado de isomaltasa.
Dcit de lactasa.
Dcit de enteroquinasa.
Gastroenteritis por rotavirus.

Lactante de un mes de edad, nacido tras un embarazo controlado


y normal, a las 35 semanas de edad gestacional, que acude a
su consulta para la revisin del nio sano correspondiente.
Todo parece estar bien salvo que los testculos se encuentran
en el canal inguinal, y aunque s lo hacen con traccin manual,
espontmente no bajan al escroto. Cul es la actitud ms adecuada en este momento?
1)

2)
3)

4)
5)

Si la alteracin es bilateral, hay que consultar con el cirujano


cuanto antes, para que realice una orquidopexia, por el riesgo
de esterilidad y degeneracin maligna.
Hay que esperar, porque es posible que espontnemente
se complete el descenso testicular.
Tratara con HCG va intramuscular a das alternos, y si no
hay respuesta, mandara al cirujano para la extirpacin de
ambos testes.
Se trata de unos testculos en ascensor, lo que no requiere
ningn tipo de tratamiento ni seguimiento.
Tratara con testosterona y HCG, y si no hay respuesta, consultara con los cirujanos para su descenso.

Tfno. (0034) 91 782 43 30/33/34 E-mail: secretaria@ctomedicina.com www. ctomedicina.com

Test

1. vuelta Distancia

Pediatra
61.

Lactante de 3 meses consulta por cuadro de vmitos y febrcula


de 48 horas de evolucin, con progresiva prdida del apetito.
En la exploracin no se objetiva foco infeccioso. Hemograma
con leucocitosis y desviacin a la izquierda. Sistemtico de
orina: leucocitos y nitritos positivos. Cul sera la actitud ms
adecuada?

tiempos de protrombina y de tromboplastina parcial activada


son normales. Ante este cuadro clnico, una de las siguientes
armaciones es cierta:
1)
2)

1)
2)
3)
4)
5)

Tratamiento con antitrmicos y ver evolucin.


Realizar puncin lumbar.
Diagnstico de infeccin de orina y administrar antibiticos
v.o.
Realizar puncin suprapbica e iniciar antibiticos.
Descartar reujo gastroesofgico.

3)
4)
5)
66.

62.

Es FALSO, respecto al reujo vesicoureteral en el nio:


1)
2)
3)
4)
5)

Puede sospecharse intratero al ver dilatacin pielocalicial


en la ecografa obsttrica.
Suele dar lugar a infecciones urinarias de repeticin.
La indicacin quirrgica parte del nmero anual de recidivas
de infeccin urinaria.
Puede ser til hacer prolaxis antibitica si se han repetido
varias infecciones urinarias.
Si el reujo es masivo, puede llevar a IRC en la infancia y
puede requerir ciruga.

Los tumores malignos son la principal causa de muerte entre


el ao y los 15 aos de edad en los pases desarrollados.
Seale la afirmacin INCORRECTA respecto a su epidemiologa:
1)

2)
3)
4)

63.

Varn de 2 aos en cuya historia clnica destaca haber padecido


un proceso diarreico en los das previos. En la exploracin se
observa palidez, estupor y hematuria. Ha presentado adems
una convulsin focal. Respecto al cuadro que usted sospecha,
seale cul de las siguientes opciones es FALSA:
1)
2)
3)
4)
5)

64.

5)

67.

2)
3)
4)
5)

En la eco-Doppler se observara disminucin del ujo sanguneo en ese testculo.


El tratamiento consiste en antibiticos, antiinamatorios, y
tras la fase aguda, ciruga.
La mayor incidencia se produce en la niez tarda y adolescencia temprana.
En mayores de 13 aos se debe hacer diagnstico diferencial
con una epididimitis.
En el periodo neonatal, generalmente tiene mal pronstico.

Un nio de 2 aos es trado a Urgencias por petequias generalizadas sin otros signos de ditesis hemorrgica. Reeren catarro
de vas altas y ebre 2 semanas antes pero en la actualidad
de encuentra afebril, con buen estado general, y el resto de la
exploracin es normal. Un hemograma muestra hemoglobina
14 mg/dl, leucocitos 9.400/mm3 y plaquetas 34.000 mm3. Los

CTO Medicina C/Francisco Silvela, 106 28002 - Madrid

4)
5)
68.

Es el tumor extracraneal ms frecuente en la infancia.


La edad ms habitual al diagnstico son los 2 aos.
La presentacin con un sndrome de mioclonus- opsoclonus
ensombrece el pronstico.
Las catecolaminas en orina estarn elevadas.
Se asocia con delecin del cromosoma 1 y anomalas del 17.

Seale la opcin FALSA respecto al tumor de Wilms:


1)
2)
3)
4)
5)

69.

La leucemia linfoblstica aguda es la neoplasia maligna ms


frecuente en la infancia, y muestra un ligero predominio en
varones.
La incidencia ms alta de enfermedad de Hodgkin se da en
adolescentes y adultos jvenes.
El neuroblastoma es el tumor slido ms frecuente en la
infancia, fuera del sistema nervioso central.
El sarcoma de Ewing es el tumor seo primario maligno
ms frecuente a partir de los 10 aos, seguido del osteosarcoma.
El rabdomiosarcoma es el sarcoma de tejidos blandos ms
frecuente en la infancia, siendo la cabeza y el cuello las
principales localizaciones.

Nia de 2 aos presenta movimientos mioclnicos de extremidades y sacudidas desordenadas de los ojos. En la exploracin
se palpa masa abdominal en lnea media y anco derecho. Se le
realiza una TC abdominal, evidencindose una masa de consistencia mixta y calcicaciones en su interior. Seale la armacin
falsa:
1)
2)
3)

Varn de 5 aos que presenta de forma sbita dolor en teste


derecho, sin antecedente de traumatismo previo. Exploracin:
afebril, teste tumefacto, doloroso a la exploracin, con ausencia
del reejo cremastrico. Seale la armacin FALSA:
1)

65.

Es la causa ms frecuente de IRA en los nios pequeos.


Existe anemia hemoltica microangioptica y trombopenia
dentro del cuadro clnico.
La trombosis bilateral de las venas renales puede provocar
un cuadro similar.
El empleo de corticoides supone el tratamiento de eleccin.
La aparicin de recidivas es muy poco frecuente.

El diagnstico ms probable es el de prpura trombtica


trombocitopnica.
Requiere de transfusin urgente de concentracin de plaquetas.
La evolucin ms probable es a la recuperacin espontnea.
Debe practicarse una TC craneal para descartar hemorragia
intracraneal.
El pronstico depende de la precocidad del tratamiento.

Se asocia a delecin del cromosoma 11, ya sea en las clulas


tumorales o en todas las clulas del organismo.
Se asocia a aniridia y anomalas genitourinarias.
La HTA es un hallazgo frecuente.
El diagnstico denitivo nos lo da la biopsia.
Suele tratarse de una masa abdominal palpable asintomtica,
con o sin metstasis pulmonares.

Un nio de 5 aos, previamente sano, debuta bruscamente con


palidez, astenia y anorexia; se le practica un hemograma y se
evidencia anemia, leucopenia y trombopenia. En la exploracin
se aprecia hepatoesplenomegalia. Qu diagnstico le parece
ms probable?

Tfno. (0034) 91 782 43 30/33/34 E-mail: secretaria@ctomedicina.com www. ctomedicina.com

Test

1. vuelta Distancia

Pediatra
1)
2)
3)
4)
5)
70.

1)
2)
3)
4)
5)

Con respecto al tumor de Wilms, es FALSO que:


1)
2)
3)
4)
5)

71.

plaquetas 71.000/ mm3. De momento no dispone de frotis de


sangre perifrica, ni de reactantes de fase aguda, pero ya tiene
una sospecha diagnstica. Seale cul es:

Anemia ferropnica.
Linfoma no Hodgkin.
Aplasia medular adquirida.
Aplasia medular congnita.
Leucemia aguda.

Puede cursar con poliglobulia.


En la UIV se observa una masa dependiente del tejido renal.
La ciruga se indica, aun en presencia de metstasis pulmonares.
Posee gran tendencia a presentar calcicaciones intraneoplsicas.
Los tumores bilaterales son ms frecuentes en los casos
familiares.

74.

4)
5)

75.
1)
2)
3)
4)
5)

72.

1)
2)
3)
4)
5)

73.

El exantema comienza por la cara, tiene evolucin descendente, y desaparece en el mismo orden en el que apareci.
La neumona de clulas gigantes de Hecht es menos frecuente que la neumona por sobreinfeccin bacteriana.
La gravedad de la enfermedad est directamente relacionada
con la intensidad y conuencia del exantema.
El prurito suele ser intenso.
Entre las posibles complicaciones se encuentra la anergia
cutnea y la reactivacin de una TBC preexistente.

Nia de 5 aos presenta cuadro catarral con ebre de 37,5


C. A la exploracin presenta adenopatas retroauriculares y
cervicales dolorosas a la palpacin e hiperemia conjuntival.
48 horas despus, aparece un exantema morbiliforme en cara.
Cul de las siguientes armaciones sobre la epidemiologa de
esta enfermedad es cierta?

76.

CTO Medicina C/Francisco Silvela, 106 28002 - Madrid

Es producida por el parvovirus B19.


El perodo de incubacin es de 7 das.
El perodo prodrmico se caracteriza por ebre alta.
El exantema caracterstico aparece en tres etapas.
Como complicacin puede producirse una crisis aplsica
grave.

Un lactante de 7 meses presenta sbitamente ebre de 40 C.


No existe ningn otro sntoma y la exploracin fsica es normal,
salvo el hallazgo de una coriza ligera y adenopatas cervicales
posteriores. El nio parece encontrarse bastante bien. Cul de
las siguientes armaciones describe mejor los factores diagnsticos que deben considerarse en este caso?
1)

2)

3)
4)

5)

77.

Est causado por un poxvirus, el virus de la varicela zster.


Las lesiones costrosas no contienen virus viables.
La complicacin ms frecuente es la sobreinfeccin de las
lesiones cutneas.
La neumona varicelosa es poco frecuente en la infancia.
La encefalitis postvaricela que se presenta con signos cerebelosos tiene mejor pronstico que si lo hace con signos
cerebrales.

Nio de 7 aos presenta rash eritematoso en ambas mejillas,


tronco y zona proximal de extremidades, con tendencia a aclararse en la zona central. Con respecto a esta enfermedad, seale
la armacin FALSA:
1)
2)
3)
4)
5)

Las personas con enfermedad subclnica no son contagiosas.


Los nios se afectan con mayor frecuencia que las nias.
Los anticuerpos maternos protegen los 6 primeros meses
de vida.
Un solo ataque no suele conferir inmunidad de duracin
permanente.
La relacin de enfermedad no aparente/maniesta es de 1
a 1.

Nio de 11 aos, que consulta por ebre, dolor de garganta


y cansancio, de 5 das de evolucin. En la exploracin fsica
destacan unas amgdalas hipertrcas con exudado en sbana,
adenopatas dolorosas axilares, inguinales, y las ms grandes,
las cervicales, y hepatoesplenomegalia de 3-4 cm por debajo
de los rebordes costales. Pide un anlisis de sangre, y lo primero que le mandan del laboratorio es el hemograma: Hb 8,1
g/dl; Hto 27%; leucocitos totales 1.800/ mm3 (600 neutrlos);

Un nio de 4 aos acude al pediatra por aparicin de lesiones


papulosas rojas, algunas con vesculas blanquecinas no umbilicadas en tronco y mucosa oral, muy pruriginosas. Dos das
antes presentaba cuadro catarral con ebre moderada. Respecto
al cuadro que presenta este nio, todo lo siguiente es cierto,
EXCEPTO:
1)
2)
3)

Nia de 6 aos que presenta desde hace 3 das ebre de 38 C


que no cede con antitrmicos, tos seca y facies congestiva. Hoy
acude por aparecer exantema maculopapuloso, rojo intenso,
no puntiforme, conuente, que no se blanquea a la presin
en la parte superior del trax, cara y parte proximal de EE.SS.
A la exploracin: T 40,3 C, adenopatas bilaterales en ngulo
mandibular y lesiones blanquecinas sobre base eritematosa en
mucosa subyugal. Respecto a la enfermedad que padece esta
paciente, seale la armacin FALSA:

Mononucleosis infecciosa.
Leucemia linfoblstica aguda.
Linfoma en fase leucmica.
Hemoglobinuria paroxstica nocturna (HPN).
Aplasia medular por antibiticos.

Si apareciera una erupcin cutnea 24 horas despus de


comenzar la ebre, sera probable el diagnstico de rosola
o exantema sbito.
En la rosola infantil, el diagnstico diferencial puede ser
difcil, puesto que en las primeras 36 horas cursa con leucocitosis, con predominio de neutrlos.
La ebre podra corresponder a los prdromos de una
rubola.
Si la ebre dura 3 das, desaparece luego rpidamente y
aparece una erupcin cutnea en ese momento, sera probable el diagnstico de eritema infeccioso.
Es probable una escarlatina, pues es una enfermedad propia
de nios de 6 a 18 meses.

Jennifer tiene 16 meses de edad y acude a Urgencias por presentar desde hace 2 das ebre de 39 C que su pediatra atribuy

Tfno. (0034) 91 782 43 30/33/34 E-mail: secretaria@ctomedicina.com www. ctomedicina.com

Test

1. vuelta Distancia

Pediatra
a cuadro respiratorio. A la exploracin presenta ebre de 40
C y contractura antilgica en exin de extremidad inferior
izquierda. Queda ingresada para estudio, y 4 das ms tarde
persiste el cuadro febril, junto con edema indurado en ambos
pies, suras labiales, conjuntivitis no purulenta y exantema
polimorfo en tronco. El diagnstico ms probable ser:
1)
2)
3)
4)
5)
78.

5)

1)
2)
3)
4)
5)

83.

3)
4)
5)

4)
5)

Hasta en el 80% hay aumento de transaminasas.


En la analtica aparece leucopenia.
Ttulos elevados de anticuerpos frente al virus de EpsteinBarr.
Existe un sndrome linfoproliferativo en varones que han
sufrido la infeccin por este virus.
La complicacin ms temida es la rotura esplnica.

Un beb de 3 meses, que recibe lactancia materna, presenta una


historia de accesos de tos con congestin facial, lagrimeo y acaba
vomitando. Hace 2 semanas present cuadro catarral de vas reas
altas. En la Rx trax se aprecian breves inltrados perihiliares. Con
respecto a este cuadro, seale la respuesta que NO sea cierta:

84.

4)
5)

2)
3)
4)
5)

Puede aparecer al principio de la vida por falta de anticuerpos


maternos.
La fase catarral precede al perodo paroxstico.
La tos paroxstica puede durar de 1 a 4 semanas.
El recuento de leucocitos suele elevarse en forma considerable con predominio de polimorfonucleares.
El perodo de incubacin oscila entre 5 y 10 das.

2)
3)

4)
5)

81.

La vacuna tipo Sabin es de virus vivos atenuados.


La vacuna oral produce inmunidad local (IgA) y general (IgG).
En el calendario actual se administra a los 2-4-6-18 meses y
entre los 4 y 6 aos.
La vacuna tipo Salk est contraindicada en familiares de
inmunodeprimidos.
En caso de diarrea o vmitos sera conveniente retrasar la
vacunacin con virus atenuados.

Seale la armacin cierta respecto a la vacuna DTP:


1)

1)

Es una vacuna de microorganismos atenuados.


La alergia al huevo ha dejado de ser una contraindicacin
absoluta para su administracin.
Est contraindicada su utilizacin en embarazadas y
pacientes VIH positivos.
Su administracin en el calendario actual se realiza a los 15
meses y a los 4 aos.
En zonas de elevada morbimortalidad puede administrarse
la primera dosis a los 6-12 meses, con una segunda dosis de
refuerzo despus del ao de vida.

Seale la armacin FALSA respecto a la vacuna de la polio:


1)
2)
3)

85.

Una IgG+ en un lactante de 10 meses indica infeccin activa


por el VIH.
La mayora de los nios infectados presenta hipergammaglobulinemia policlonal precoz.
La linfopenia e inversin del cociente CD4/CD8 son menos
llamativas que en el adulto.
No es til la determinacin de IgA contra el VIH, porque no
suele aparecer hasta el 6. mes de vida.
Los resultados positivos en la PCR han mostrado una correlacin positiva con el aislamiento del virus en cultivo.

Respecto a la vacuna triple vrica, seale la armacin FALSA:


1)
2)

Reaccin a la penicilina.
Prpura de Schnlein-Henoch.
Mononucleosis infecciosa.
Enfermedad de Kawasaki.
Prpura trombocitopnica idioptica.

La neumona por MAI es la infeccin oportunista ms frecuente en el SIDA peditrico.


La manifestacin ms frecuente en el SIDA infantil es la
hepatitis B.
La parotiditis es ms frecuente en el SIDA infantil que en el
del adulto.

Sobre el diagnstico de un nio VIH positivo, es FALSO que:

Respecto a la mononucleosis infecciosa, es FALSO que:


1)
2)
3)

80.

4)

82.

Paciente varn de 5 aos de edad que acude por presentar


cuadro febril y odinofagia con exudado blanquecino en ambas
amgdalas que su pediatra trat con penicilina oral. Dos das
despus, presenta petequias y prpura palpables, principalmente en miembros inferiores, as como artralgias en ambos
tobillos que le impiden la marcha y dolor abdominal de tipo
clico. Los estudios complementarios revelan 13 g/dl de Hb;
leucocitos 10.500/mm3 con frmula normal; plaquetas 485.000/
mm3; tiempo de protrombina de 95% y tiempo parcial de tromboplastina de 27 segundos (control 25 segundos). La causa ms
probable de estos hallazgos es:
1)
2)
3)
4)
5)

79.

Rubola.
Mononucleosis infecciosa.
Prpura de Schnlein-Henoch.
Enfermedad de Kawasaki.
Eritema infeccioso.

3)

La vacuna de la tos ferina est contraindicada en mayores


de 18 meses.
No puede administrarse conjuntamente con otras vacunas.
La vacuna DTPa (acelular) tiene mayor riesgo de efectos
secundarios que la DTP, por lo que no se utiliza de forma
rutinaria.
A los 14 aos es preciso vacunar con Td (ttanos y difteria
adultos), y no son necesarias nuevas dosis de recuerdo.
No est contraindicada en pacientes inmunodeprimidos.

Con respecto al SIDA en la infancia, seale la opcin correcta:


86.
1)
2)

El SIDA precoz es menos frecuente que el tardo y la clnica


predominante es infecciosa.
El SIDA tardo es ms frecuente y predomina la clnica neurolgica.

CTO Medicina C/Francisco Silvela, 106 28002 - Madrid

Con respecto a la vacuna contra Haemophilus inuenzae tipo B,


es FALSO que:
1)

Las nuevas vacunas conjugadas son aptas para la inmunizacin en lactantes mayores de 2 meses.

Tfno. (0034) 91 782 43 30/33/34 E-mail: secretaria@ctomedicina.com www. ctomedicina.com

10

Test

1. vuelta Distancia

Pediatra
2)
3)
4)
5)

87.

2)

3)
4)
5)

4)
5)

93.

Realizar quimioprolaxis con ceftriaxona durante 2 das.


No existe quimioprolaxis ecaz.
Administracin de la vacuna antimeningoccica en nios
menores de 5 aos.
Administracin de rifampicina durante 2 das.
Administracin de rifampicina durante 4 das.

3)
4)
5)

Observacin y repetir Mantoux en 6 meses.


Realizar quimioprolaxis primaria durante 2 meses con INH
y repetir el Mantoux posteriormente.
Realizar quimioprofilaxis primaria durante 6 meses con
INH.
Administrar tratamiento completo con tres frmacos durante
6 meses.
Vacunacin con BCG.

94.

2)
3)
4)
5)

95.
1)
2)
3)
4)
5)

Antecedentes familiares.
Prematuridad.
Lactancia materna.
El sexo masculino.
El tabaquismo materno.

1)
2)

La persistencia del ductus arterioso se asocia a todo lo siguiente,


EXCEPTO a:
1)
2)
3)
4)
5)

Pulsos perifricos saltones.


Mayor frecuencia en prematuros.
Soplo continuo en el rea pulmonar.
Puede ser efectivo el tratamiento con indometacina.
Las prostaglandinas son el mejor tratamiento mdico para
conseguir su cierre.

CTO Medicina C/Francisco Silvela, 106 28002 - Madrid

4)
5)

96.

El tipo ms frecuente es la coartacin de localizacin yuxtaductal.


La mayor parte de los casos se muestran asintomticos.
La tensin arterial en los miembros inferiores es mayor que
en los superiores.
En ocasiones, la tensin arterial en el brazo derecho es mayor
que en el izquierdo.
En la radiografa de trax, con el tiempo, se visualizan escotaduras en los bordes inferiores de las costillas.

Con respecto al desarrollo puberal normal, es verdadero:

3)
91.

Meningitis bacteriana.
Malos tratos.
Enfermedad metablica no diagnosticada descompensada.
Hipoglucemia.
Hiponatremia.

Con respecto a la coartacin de aorta (CoA), seale la FALSA:


1)

Respecto al sndrome de muerte sbita infantil o del lactante


(SMSL), seale cul de los siguientes NO constituye un factor
de riesgo:

Tetraloga de Fallot.
Coartacin de aorta.
Anomala de Ebstein.
Transposicin simple de las grandes arterias.
Atresia tricuspdea.

Lactante de 9 meses de edad que llega a Urgencias trado por


su madre, por somnolencia de 12 horas de evolucin. No ha
tenido ebre, vmitos, ni ninguna otra sintomatologa. En la
actitud de la madre destaca que parece darle poca importancia
al estado del nio, contestando a las preguntas de la anamnesis con desgana, aunque espontneamente expresa todas
las dicultades por las que est pasando para compaginar su
papel de madre con su trabajo. En la exploracin fsica, el nio
est consciente y reacciona con irritabilidad a estmulos, pero
tiende a quedarse dormido. Tiene un hematoma periorbitario
reciente, y varios pequeos antiguos en la regin dorsolumbar,
que la madre atae a cadas accidentales, aunque el nio est
iniciando ahora la bipedestacin con ayuda. Qu sospechara
inicialmente como causante del estado del nio?
1)
2)
3)
4)
5)

Cul sera la actitud a seguir ante un nio de 4 aos asintomtico


con Mantoux negativo, hijo de un tuberculoso bacilfero?
1)
2)

90.

La vacuna de VHB est realizada por ingeniera gentica con


efectos adversos mnimos.
La pauta de administracin en caso de riesgo inmediato
como contacto con una jeringuilla sera 0-1-2 meses, y al
ao.
La vacuna VHB no est recomendada en mujeres embarazadas.
No est indicada su administracin en nios mayores de 7
aos.
Se puede administrar con la gammaglobulina anti-VHB,
siempre que se administre en distinto punto.

Ante un recin nacido de 2 das con cianosis, taquipnea sin


soplo a la auscultacin y que, en la radiografa de trax,
muestra ligera cardiomegalia, con un pedculo cardaco
estrecho y aumento del flujo sanguneo pulmonar, el diagnstico ms probable es:
1)
2)
3)
4)
5)

Cul sera la actitud ms adecuada a seguir en los contactos


de un nio con meningitis meningoccica?
1)
2)
3)

89.

92.

Seale la armacin FALSA respecto a la vacuna del VHB:


1)

88.

Es muy inmungena.
Sus efectos adversos son mnimos.
Es necesario aplicarla despus de los 5 aos.
En el calendario vacunal debera administrarse a los 2-4-6 y
18 meses.

El primer dato a la exploracin sugestivo de inicio de la


pubertad en las nias es la pubarquia.
El aumento del tamao testicular en el varn es el primer
signo de pubertad.
La pubertad suele iniciarse a los 8 aos en las nias y a los
9 en los nios.
El vello sexual es el primer signo de pubertad.
La ganancia de peso y talla es ms acusada en las nias que
en los nios.

Nia de 4 aos que consulta por desarrollo mamario lentamente


progresivo desde hace 2 meses, sin adrenarquia ni aumento de la
velocidad de crecimento. Los padres no lo relacionan con ningn
desencadenante ni con otros signos-sntomas. La exploracin

Tfno. (0034) 91 782 43 30/33/34 E-mail: secretaria@ctomedicina.com www. ctomedicina.com

11

Test

1. vuelta Distancia

Pediatra
fsica general es normal, con un estadio puberal II de Tanner (T2
P1 Aa). Se realiza una radiografa de mano y mueca izquierdas,
estimndose la edad sea en 3,5-4 aos. En la ecografa plvica
se visualizan tero y ovarios de morfologa y tamao adecuados
a la edad de la paciente, sin quistes signicativos. Los niveles
sricos de estradiol son de 40 pg/dl (N 10-30), y la FSH y la LH
basales son normales. Cul es su diagnstico de sospecha?
1)
2)
3)
4)
5)

97.

Cul de los siguientes cuadros no forma parte de diagnstico


diferencial de talla baja en los nios?
1)
2)
3)
4)
5)

98.

Sndrome de Turner.
Sndrome de Down
Sndrome de Silver-Russell.
Sndrome de Klinefelter.
Sndrome de Noonan.

Respecto a las convulsiones neonatales, seale la armacin


FALSA:
1)
2)
3)
4)
5)

99.

Pubertad precoz central, probablemente idioptica.


Pubertad precoz central, probablemente por hamartoma
hipotalmico.
Telarquia prematura aislada.
Pubertad precoz perifrica.
Aunque no sepamos el ndice de masa corporal (IMC) de la
paciente, lo ms probable es que sea elevado y nos encontremos realmente ante una adipomastia bilateral.

Los RN con convulsiones suelen tener mal pronstico.


El tratamiento consiste en medidas de sostn y fenobarbital
o diazepam.
Las crisis tnico-clnicas generalizadas son raras durante el
primer mes de vida.
La clnica es variable y el EEG puede ser la nica forma para
su diagnstico.
La causa ms frecuente de crisis en el perodo neonatal es
la encefalopata hipoxicoisqumica.

Acerca de la enuresis, seale cul es la respuesta correcta:


1)
2)
3)
4)
5)

Es ms frecuente en el sexo femenino.


La enuresis secundaria es ms frecuente entre los 5 y los 8
aos de edad, y tiene peor pronstico que la primaria.
La predisposicin gentica es un factor de riesgo muy
importante para la enuresis nocturna.
La causa ms frecuente de enuresis diurna es la infeccin
de orina.
El tratamiento de eleccin en la enuresis nocturna persistente
es la desmopresina administrada antes de acostarse.

100. Llega a Urgencias un nio de 18 meses con ebre de 39 C, que


ha presentado en su domicilio un episodio tnico-clnico generalizado de 3 minutos de duracin, con posterior somnolencia.
Lo correcto a su llegada a Urgencias sera:
1)
2)
3)
4)
5)

Administracin de bolo de glucosa.


Administracin de antitrmicos y diazepam.
Administracin de fenitona.
Realizar puncin lumbar.
Administracin de piridoxina.

CTO Medicina C/Francisco Silvela, 106 28002 - Madrid

Tfno. (0034) 91 782 43 30/33/34 E-mail: secretaria@ctomedicina.com www. ctomedicina.com

12

Comentarios de Test a distancia 1. vuelta

Pediatra

Comentarios de Test a distancia

1. vuelta

Pediatra
Pregunta 1.- R: 3
El test de Apgar se debe realizar a todo recin nacido, independientemente
de su edad gestacional sano o con sospecha de patologa (respuesta 1
falsa). Se realiza al minuto y a los 5 minutos y posteriormente slo se
repetir cada 5 minutos si la puntuacin a los 5 minutos es inferior de
7 (respuesta 3 correcta).
Entre los parmetros que valora se encuentran: color, frecuencia cardaca,
respuesta a la estimulacin, tono muscular y esfuerzo respiratorio. NO
frecuencia respiratoria (respuesta 2 falsa).
A pesar de una puntuacin baja en los primeros minutos de vida, no
sirve para establecer valoraciones pronsticas (respuesta 4 falsa). Una
puntuacin inferior a 3, mantenida ms all de los 20 minutos de vida,
s puede predecir una elevada morbimortalidad.
La palidez cutnea generalizada supone una puntuacin de 0 en el test
de Apgar (respuesta 5 falsa).
Pregunta 2.- R: 4
Si seguimos la tabla del comentario de la pregunta anterior obtendremos
una puntuacin de 4 en el test de Apgar realizado a este recin nacido.
Pregunta 3.- R: 1
La alimentacin de los prematuros debe realizarse de forma cautelosa.
Siempre que no haya distrs respiratorio ni otras circunstancias que
contraindiquen su inicio, hay que procurar una alimentacin precoz
para evitar la hipoglucemia, la deshidratacin y la hiperbilirrubinemia. Se
puede intentar va oral y continuar si hay buena tolerancia, aunque en los
menores de 32 semanas es muy frecuente que no exista la coordinacin
adecuada y haya que colocar una SNG (respuesta 1 falsa).

Esfuerzo respiratorio
Respuesta
a la introduccin
de una sonda nasogstrica
Frecuencia cardaca
Tono
muscular
Color

La anemia siolgica, aparece antes y con ms intensidad que en los


nacidos a trmino, por los menores depsitos de hierro y el rpido
crecimiento, por lo que a veces precisan ser tratados con eritropoyetina, que debe acompaarse siempre de suplementos de hierro va oral
(respuesta 2 correcta).
Las prdidas insensibles de lquido estn aumentadas respecto a los
neonatos nacidos a trmino, en relacin inversa a la edad gestacional,
y sus necesidades diarias son mayores (respuesta 3 correcta).
En todos los recin nacidos, pero ms en los prematuros, los txicos
de eliminacin renal, y en los prematuros tambin los de eliminacin
heptica, deben darse en menor dosis o con intervalos mayores (respuesta 4 correcta).
En lo RNPT, la hipoxia produce afectacin de la matriz germinal, estructura
transitoria presente en el cerebro inmaduro hasta la semana 34 de gestacin. Dado que est compuesta por vasos su lesin produce hemorragia.
Hay que realizar sistemticamente ecografa craneal a todos los recin
nacidos < 32 SG y a los recin nacidos > 32SG con factores de riesgo como
asxia perinatal o signos neurolgicos anormales (respuesta 5 correcta).
Pregunta 4.- R: 3
La respuesta 1 es cierta: es importante para el MIR tener claro las diferencias
entre caput (edema de partes blandas, no limitado por suturas) y cefalohematoma (hematoma entre hueso y periostio, limitada por suturas).
La respuesta 2 es cierta: el traumatismo obsttrico ms frecuente es la
fractura de clavcula. A la exploracin lo ms llamativo es la crepitacin
y el reejo de Moro asimtrico.
La respuesta 3 es falsa: el llamado ndulo de Stromayer es un hematoma
palpable a nivel del ECM. La mayora de las veces se reabsorbe espont-

Ausente

Lento, irregular

Bueno, llanto

Sin respuesta

Mueca

Tos o estornudo

Ausente

Menos de 100

Ms de 100

Dbil

Ligera exin
de extremidades

Movimientos activos

Azul, plido

Cuerpo sonrosado,
extremidades cianticas

Totalmente sonrosado

Pregunta 1. Test de Apgar.


CTO Medicina C/Francisco Silvela, 106 28002 - Madrid

Tfno. (0034) 91 782 43 30/33/34 E-mail: secretaria@ctomedicina.com www. ctomedicina.com

Comentarios de Test a distancia 1. vuelta

Pediatra
neamente, pero en ocasiones se puede brosar acortando el msculo.
El tratamiento se basa en la sioterapia y muy rara vez requiere ciruga.
La respuesta 4 es cierta: la vscera que con mayor frecuencia se lesiona es
el hgado, pero la mayora de las veces son pequeos hematomas subcapsulares limitados que, secundariamente, pueden producir ictericia
precoz prolongada, sin otra repercusin clnica. Recordar para el MIR el
cuadro clnico tpico de la hemorragia suprarrenal: nio grande, parto de
nalgas, que en las primeras horas de vida presenta clnica de hipotensin,
sangrado y shock.
La respuesta 5 es cierta: las fracturas craneales lineales generalmente
no precisan tratamiento quirrgico, salvo que se traten de fracturas
deprimidas que produzcan lesiones en el parnquima.
Pregunta 5.- R: 4
Dentro de los fenmenos dermatolgicos que podemos encontrar en un
neonato, y que no tienen signicacin patolgica, se hallan: los quistes
de milium, la mancha monglica, los angiomas planos, el eritema txico
y la melanosis pustulosa.
El eritema txico es ms propio de la raza blanca y consiste en vesiculopstulas sobre base eritematosa que suele respetar palmas y plantas y tiende a
desaparecer en la primera semana. La melanosis pustulosa es ms frecuente
en la raza negra, la erupcin vesiculopustulosa s suele afectar a las palmas
y plantas, y tiene tendencia a desaparecer en varias semanas. Recuerda las
principales diferencias entre eritema txico y la melanosis pustulosa.
ERITEMA TXICO

MELANOSIS PUSTULOSA

Aparicin

1-3 das

Nacimiento

Localizacin

Variable
No palmoplantar

Variable
S palmoplantar

Frotis

Eosinlos

Neutrlos

Cultivo

Estril

Estril

Pregunta 5. Melanosis pustulosa vs. eritema txico del RN.


Las manchas hipopigmentadas geogrcas se asocian a esclerosis
tuberosa (respuesta 1).
La ictericia en las primeras 24 horas de vida es siempre patolgica (respuesta 2) por hemlisis e infecciones (sepsis, TORCH).
El nevus ammeus o manchaen vino de Oportose asocia a angiomatosis
encefalotrigeminal o sndrome de Sturge-Weber (respuesta 3).
Manchas caf con leche se asocian a neurobromatosis tipo 1 (respuesta 5).
Pregunta 6.- R: 2
El cuadro clnico se reere a una taquipnea transitoria del RN, sndrome
de Avery, pulmn hmedo o SDR tipo II.
La presentacin tpica es un RN nacido por cesrea o por parto vaginal
rpido, lo que supone que el trax no ha estado sometido a la presin
positiva que exprime de lquido los alveolos del pulmn. La clnica se
inicia en las primeras horas de vida, que cede y mejora con medidas
poco agresivas (oxgeno en incubadora). Suelen alcanzar la resolucin
clnica en un plazo mximo de 3 das.
Lo que establece el diagnstico, aparte de la evolucin clnica, es la Rx de
trax, donde se aprecia aumento de marcas vasculares, lquido en las cisuras.
La EMH es tpica de prematuros, especialmente habitual en hijos de
madre diabtica, por dcit de surfactante y por inmadurez estructural
pulmonar. Rx de trax: patrn reticulonodular (vidrio esmerilado) con
broncograma areo (respuesta 1 falsa).
El SAM es tpico de RN postrmino, con antecedentes de estrs en
el momento del parto. Rx trax: inltrados parcheados algodonosos
(respuesta 3 falsa).
CTO Medicina C/Francisco Silvela, 106 28002 - Madrid

La PCF suele cursar con clnica marcada de cianosis que no responde a la administracin de oxgeno. No hay una Rx de trax caracterstica (respuesta 4 falsa).
La DBP es tpica de pretrminos que han necesitado oxgeno con el
antecedente de barotrauma o EMH. Rx de trax: patrn en esponja.
(respuesta 5 falsa).
Pregunta 7.- R: 2
Estamos ante un cuadro tpico de enfermedad de membrana hialina: RNPT
con dicultad respiratoria intensa que aparece en las primeras horas de
vida, que no mejora tras la administracin de oxgeno. En la gasometra
es llamativa la importante hipoxia. Para el diagnstico es necesario realizar
una Rx de trax, donde lo ms tpico es encontrar un inltrado reticulonodular (en vidrio esmerilado) con broncograma areo. La imagen radiolgica no es patognomnica (tambin aparece en neumonas).
Respuesta 3: corresponde a la taquipnea transitoria del RN.
Respuesta 4: tpico de la displasia broncopulmonar.
Respuesta 5: corresponde a un neumotrax bilateral.
Pregunta 8.- R: 4
El tratamiento de la EMH consiste en:
Monitorizacin estrecha, control de lquidos.
Intentar mantener una aceptable oxigenacin (pO2 50-70 mmHg). Si no
se consigue con oxgeno indirecto, ser necesario intubar al paciente e
iniciar ventilacin mecnica.
Administracin de surfactante intratraqueal en las primeras 24 horas de
vida, pudiendo recibir hasta 4 dosis segn evolucin. Esta medida se ha
visto que mejora la clnica de la EMH, pero que no disminuye los casos
de DBP.
Administracin de antibiticos: ampicilina ms gentamicina, porque
la clnica y la Rx pueden ser semejantes en el caso de una sepsis con
neumona.
La respuesta incorrecta es la 4. Las prostaglandinas en los neonatos
estn indicadas en caso de cardiopatas ductus dependientes, en las
que es IMPRESCINDIBLE mantener el ductus abierto.
Pregunta 9.- R: 2
El caso clnico es el de un nio que ha sufrido un SAM. Lo tpico es que se
produzcan en RN postrmino, con un estrs en el momento del parto, que
estimula el peristaltismo intestinal y se produce la eliminacin de meconio
intratero.
Al nacimiento, el meconio se encuentra en las vas areas altas; si no se elimina
a tiempo, ste pasar al pulmn con las primeras respiraciones, formando
tapones que ejercen mecanismo valvular favoreciendo el atropamiento
de aire y produciendo un cuadro de alveolitis con riesgo de atelectasias y
sobreinfeccin bacteriana, siendo el germen ms frecuente E. coli. En los
casos ms graves se asocia con una alta mortalidad debido a la hipertensin
pulmonar que desarrollan.
El tratamiento consiste en ventilacin mecnica, antibiticos prolcticos y medidas para disminuir la hipertensin pulmonar: alcalinizar,
tolazolina y oxido ntrico inhalado. Algunos nios se pueden beneciar
de la ECMO (oxigenacin por membrana extracorprea).
Pregunta 10.- R: 1
Caso clnico de una parlisis braquial superior, tambin conocida como parlisis de Erb-Duchenne. Se debe a la afectacin de las races C5-C6 del plexo
braquial. El recin nacido presenta el brazo en aduccin y rotacin interna
con el reejo de Moro ausente en el lado de la lesin (respuesta 1 correcta).
Sin embargo, el reejo de prensin palmar suele estar presente al no verse
afectadas las races inferiores. En los casos en que se lesione tambin la raz
C4 puede asociarse con una parlisis frnica.

Tfno. (0034) 91 782 43 30/33/34 E-mail: secretaria@ctomedicina.com www. ctomedicina.com

Comentarios de Test a distancia 1. vuelta

Pediatra
PARLISIS BRAQUIAL

P. ERB-DUCHENNE

P. KLUMPKE

Races

(C4) - C5 - C6

C7 - C8 - (T1)

Clnica

Brazo en aduccin
y rotacin interna

Mano cada

R. Moro

No presente o asimtrico

Presente

R. prensin palmar

Presente

No presente

Asociaciones

C4 - Parlisis frnica

T1 - S. Horner

Pregunta 10. Parlisis braquial.


Pregunta 11.- R: 5
Estamos ante un caso tpico de displasia broncopulmonar: nio de
un mes de vida con antecedente de prematuridad y enfermedad de
membrana hialina que requiere altas concentraciones de oxgeno para
mantener adecuadas saturaciones. Dentro de los factores de riesgo para
este cuadro guran: la toxicidad del oxgeno en altas concentraciones,
inmadurez pulmonar y barotrauma.
El diagnstico se establece por la historia clnica y la Rx de trax, donde
es tpico encontrar el llamado patrn de esponja (reas ms claras que
alternan con otras de mayor densidad).
El tratamiento se basa en oxigenoterapia, restriccin de lquidos y diurticos, broncodilatadores y corticoides (slo en casos graves). Se ha
demostrado que la vitamina A disminuye la incidencia de DBP.
La mayora de los nios suelen tener un curso favorable, alcanzando
la normalidad de la funcin pulmonar hacia los 2 aos de vida. Hay un
pequeo porcentaje que evoluciona hacia un cuadro de hipertensin
pulmonar persistente (respuesta 5 falsa). Las dos causas ms frecuentes de
muerte en estos nios son la ICC derecha y la bronquiolitis necrotizante.
Pregunta 12.- R: 3
Se trata de un caso tpico de hernia diafragmtica congnita. La posterior
o de Bochdaleck es el tipo de hernia diafragmtica ms frecuente, sobre
todo izquierda. Consiste en un oricio grande en la parte posterior izquierda del diafragma a travs del que vsceras abdominales migran hacia el
trax, con lo que el abdomen aparece excavado (vaco de contenido) y hay
un compromiso respiratorio grave secundario a la hipoplasia y compresin
pulmonar. Es un cuadro severo que aboca a la instauracin de hipertensin pulmonar que ya est presente en el nacimiento (respuesta 5 falsa).
El diagnstico en la mayora de los casos se hace por ecografa prenatal.
Cuando no es as, se realiza Rx de trax al nacimiento donde advertiremos la presencia de asas intestinales en trax, y el mediastino estar
desplazado por el efecto masa de stas. Si tras la radiografa hay dudas
con malformaciones pulmonares qusticas, hay que realizar una ecografa
(respuesta 3 correcta).
El cierre quirrgico se har a las 24-72 horas tras estabilizar la situacin
hemodinmica del paciente, primero durante 10-12 horas forzaremos
una situacin de alcalosis (el pH alcalino vasodilata las arterias pulmonares) para minimizar la hipertensin pulmonar (respuesta 2 falsa).
Prcticamente todos tienen reujo gastroesofgico al nacer, y hasta un
20% necesita una funduplicatura (respuesta 4 falsa).
La hernia diafragmtica congnita es ms frecuente en algunos sndromes
congnitos, como las trisomas 21, 13 y 18. En un 20-30% de los casos
hay otras alteraciones asociadas (respuesta 1 falsa) como malrotacin
intestinal (la ms frecuente), cardiopatas y malformaciones vasculares.

La matriz germinal es una estructura transitoria, presente en el cerebro


inmaduro en la cabeza del ncleo caudado, cerca de los ventrculos laterales hasta la semana 34 de gestacin y es muy sensible a la hipoxia. Dicha
matriz est constituida por vasos y su lesin produce hemorragia que se
diagnostica a travs de una ecografa transfontanelar (respuesta 3 correcta).
Pregunta 14.- R: 4
Estamos ante un RN con un cuadro de enterocolitis necrotizante (NEC).
La presentacin tpica es en un RNPT que a la segunda semana de vida
inicia cuadro de vmitos, distensin abdominal y deposiciones con sangre.
Dentro de los factores de riesgo guran, aparte de la prematuridad, la
administracin precoz y con altas concentraciones de leche, situaciones
de hipoxia y bajo gasto y todos aquellos cuadros que favorezcan isquemia
intestinal. La prueba diagnstica indicada sera una Rx de abdomen
donde se apreciara el dato ms caracterstico, la neumatosis intestinal.
El tratamiento se debe instaurar ante la mnima sospecha, puesto que
de eso va a depender el pronstico. Recuerda que la lactancia materna
es factor protector. Si el nio no tiene signos de perforacin intestinal,
el tratamiento ser dieta absoluta y antibioterapia que cubra anaerobios
y gramnegativos. El tratamiento ser quirrgico en caso de perforacin
intestinal o sepsis refractaria al tratamiento mdico.
Las complicaciones a largo plazo son la estenosis intestinal y el sndrome
de intestino corto.
Pregunta 15.- R: 3
El cuadro clnico se reere a un leo meconial. La forma de presentacin
ms habitual en los neonatos es como un cuadro de obstruccin intestinal congnita. El dato que apoyara este diagnostico sera palpar unos
cordones duros que siguen el marco clico, puesto que el lugar donde
con mayor frecuencia se impacta el meconio es la zona de leon distal y
coln proximal. El diagnstico se hace mediante radiografa de abdomen
con distensin de asas intestinales y patrn granular espumoso en los
puntos de concentracin meconial.
Una vez establecido el diagnstico es siempre obligatorio descartar una
brosis qustica (un 15% debutan como leo meconial).
El tratamiento, si el nio no tiene signos de perforacin intestinal, se
realiza mediante la administracin de enemas hiperosmolares. Si con
esta medida no se soluciona el problema o se producen complicaciones,
sera necesario recurrir a la ciruga, realizando una reseccin de la zona
de impactacin y anastomosis terminoterminal.
Pregunta 16.- R: 3
Respecto a las ictericias neonatales, es importante tener claro varios
aspectos.
A continuacin se reejan las causas ms frecuentes de ictericia
segn el momento de aparicin:
1.as 24 horas Hemlisis.
Infecciones: sepsis, TORCH.
Fisiolgica.
2.-3.er da
Infecciones: sepsis, TORCH.
Anemias hemolticas.
4.-7. da

Sepsis.
TORCH.
Obstruccin intestinal.
Lactancia materna.

>1 mes
Pregunta 13.- R: 3
Estamos ante caso tpico de hemorragia de la matriz germinal: RNPT
con distrs respiratorio y que de forma brusca desarrolla hipotensin,
bradicardia, cianosis, fontanela abombada y disminucin del hematocrito,
el diagnostico ser de hemorragia de la matriz germinal.
CTO Medicina C/Francisco Silvela, 106 28002 - Madrid

Galactosemia, hipotiroidismo, lactancia materna,


metabolopatas, ictericia obstructiva, Gilbert,
Crigler-Najjar.
Caractersticas de la ictericia siolgica vs. no siolgica:
- Inicio en las primeras 24 horas: SIEMPRE patolgico.
- Predominio de bilirrubina directa: SIEMPRE patolgico.

Tfno. (0034) 91 782 43 30/33/34 E-mail: secretaria@ctomedicina.com www. ctomedicina.

Comentarios de Test a distancia 1. vuelta

Pediatra

Otros datos de la ictericia no siolgica:


- Incremento mayor de 5 mg/24 horas.
- Bilirrubina en sangre de cordn mayor de 3 mg/dl.
- RNT > 12 y RNPT > 14.
- Duracin superior a 14 das.

Pregunta 17.- R: 3
La ictericia por lactancia materna (LM) es la causa ms frecuente de ictericia
tarda. Su frecuencia es de aproximadamente 1/200 (respuesta 1 correcta).
Comienza a manifestarse entre el 5. y el 7. da de vida alcanzando su
mximo en la tercera semana (respuesta 2 correcta).
Se debe a que en la leche materna existen unas sustancias (pregnanodiol,
cidos grasos de cadena larga) que inhiben la glucuronil-transferasa,
dando como resultado un aumento de la bilirrubina a expensas de
fraccin indirecta (respuesta 4 correcta).
En ocasiones, cuando existen dudas diagnsticas, se puede interrumpir la
LM transitoriamente durante unos das, observndose un claro descenso
en la bilirrubina, pero sta no es indicacin de suspender denitivamente
la LM, puesto que suele ser moderada (valores inferiores a 15) y no se
han descrito casos de kernicterus (respuesta 3 falsa).
Pregunta 18.- R: 5
Para el MIR es importante saberse bien las diferencias entre la incompatibilidad Rh y AB0.
Rh madre (-), hijo (+)

AB0 madre (0)

Frecuencia

Menor

Mayor

Aparicin

Despus del primer embarazo

Primer embarazo

Gravedad

Ms grave (ictericia,
anemia)
Hidrops

Leve (ictericia, anemia)


No hidrops

Diagnstico

Coombs directo +; indirecto +

Coombs directo +/-;


indirecto +

Tratamiento

Exanguinotransfusin/
fototerapia
Fototerapia y/o
Prevencin
exanguinotransfusin
con gammaglobulina anti-D

Pregunta 18. Diagnstico diferencial de la isoinmunizacin


del recin nacido.
Pregunta 19.- R: 4
El caso clnico hace referencia a una enfermedad hemorrgica del RN. La
causa es un dcit de vitamina K que produce a su vez dcit de factores
k dependientes (respuesta 1 correcta), debido a que durante el embarazo el paso transplacentario de vitamina K es escaso. La LM es pobre
en vitamina K (respuesta 4 falsa) y en el intestino no hay bacterias que
sinteticen esta vitamina. Todas estas circunstancias obligan a administrar
vitamina K (1 mg i.m.) a todos los RN en las primeras 24 horas de vida.
La clnica consiste en sangrado a distintos niveles en las primeras 24
horas (respuesta 2 correcta): umbilical, digestivo, nasal, etc.
El tratamiento consiste en la administracin de nuevas dosis de vitamina
K o plasma fresco congelado (respuesta 5 correcta).
Es importante recordar que los hijos de madres que durante el embarazo
han tomado fenitona o fenobarbital tienen ms riesgo de padecer este
trastorno (respuesta 3 correcta).
Pregunta 20.- R: 1
La llamada anemia siolgica se debe a un dcit transitorio de EPO,
que unido a la hemlisis de glbulos rojos durante los primeros meses
de vida, dejan al nio anmico (respuesta 2 correcta).
CTO Medicina C/Francisco Silvela, 106 28002 - Madrid

Las cifras ms bajas se producen a los 2 meses en los RNPT y a los 3 en


los RNT (respuesta 1 falsa).
El tratamiento consiste en suplementos de hierro a partir de los 2 meses
en los RNPT, ya que antes los depsitos estn llenos por la hemlisis
de los glbulos rojos (respuesta 3 correcta), y en casos seleccionados
sera necesario la transfusin de concentrados de hemates (respuesta
4 correcta).
En los neonatos no se maniesta la betatalasemia, porque la cadena
beta de la hemoglobina se empieza a sintetizar a partir de los 6 meses
(respuesta 5 correcta).
Pregunta 21.- R: 3
Es muy importante recordar las situaciones en las que se debe administrar gammaglobulina especca anti-D para prevenir la isoinmunizacin
Rh. Es necesario inyectar la gammaglobulina anti-D a las 28 semanas
de gestacin y en las primeras 72 horas despus del parto, aborto o
amniocentesis, si se conrma que el recin nacido es Rh positivo y la
madre Rh negativo. La prolaxis se har slo si la gestante no ha sido
previamente sensibilizada, es decir, slo si el test de Coombs indirecto
de la gestante es negativo (respuesta 3 falsa).
Pregunta 22.- R: 5
La hipoglucemia neonatal es bastante frecuente en los hijos de madre
diabtica (75% de los hijos de madres diabticas y 25% de los hijos de
madres con diabetes gestacional). Se debe a que el HMD sintetiza exceso de insulina para compensar el dcit materno de esta hormona y
el riesgo es proporcional a los niveles de glucosa en sangre de cordn.
Recuerda que puede aparecer en las primeras 48 horas de vida, riesgo
mximo entre las 3 y las 6 horas de vida y normalmente cursa de forma
asintomtica.
El control durante el parto de la glucemia materna junto con el control
de la glucemia venosa o capilar en el neonato y el inicio precoz de la alimentacin oral (respuesta 5 falsa), disminuyen el riesgo de hipoglucemia
neonatal. El tratamiento consiste en el aporte de glucosa i.v. mediante
perfusin continua, evitando los bolos por el riesgo de hiperglucemia
de rebote.
Pregunta 23.- R: 1
El embarazo del hijo de madre diabtica es considerado un embarazo de
alto riesgo, debido al gran nmero de problemas que puede presentar
el RN. Entre ellos guran:
Mayor mortalidad fetal y neonatal.
Polihidramnios.
Macrosoma con visceromegalia si la madre no tiene vasculopata,
si la tiene, CIR.
Estenosis subartica con hipertroa septal asimtrica.
Riesgo de EMH: la insulina en altas concentraciones inhibe la sntesis
de surfactante.
Policitemia y sus consecuencias (ictericia y trombosis de la vena renal).
Mayor incidencia de malformaciones congnitas:
- Malformaciones ms frecuentes: cardacas.
- Malformacin digestiva ms frecuente: colon izquierdo hipoplsico.
- Malformacin ms caracterstica: agenesia lumbosacra.
Alteraciones metablicas:
- Hipoglucemia: mxima en las 3-6 primeras horas.
- Hipocalcemia.
Pregunta 24.- R: 3
Recuerda las principales caractersticas de la sepsis PRECOZ, TARDA y
NOSOCOMIAL.

Tfno. (0034) 91 782 43 30/33/34 E-mail: secretaria@ctomedicina.com www. ctomedicina.com

Comentarios de Test a distancia 1. vuelta

Pediatra
ETIOLOGA

CLNICA

Sepsis precoz
(3. a 5. da)

S. agalactiae
E. coli
Listeria monocytogenes

Sepsis tarda
(7-28 das)

S. agalactiae serotipo III


E. coli serotipo K1

Sepsis
nosocomial

S. epidermidis
S. aureus
P. aeruginosa
C. albicans

CURSO

PRONSTICO

Ms afectacin respiratoria
(quejido, polipnea, tiraje,
aleteo, cianosis), ictericia
Listeria: adems,
granulomas en faringe,
petequias

Fulminante

Muy malo
(mueren 30%)
La listeriosis es muy
grave (mueren
40-80%)

Ampicilina + gentamicina
(ampicilina + cefotaxima,
si meningitis)

Ms afectacin del SNC


(meningitis neonatal)
Focalizan ms

Menos
fulminante

Menos malo
Alta morbilidad

Ampicilina + gentamicina
Si meningitis:
- No ingresados: ampicilina
+ cefa 3.

Variable

Vancomicina + amikacina
+ anfotericina B
Si meningitis:
vancomicina + ceftacidina
+ anfotericina B

Variable

Variable

TRATAMIENTO

Pregunta 24. Sepsis neonatal.


Pregunta 25.- R: 5
La madre que puede contagiar una hepatitis B al RN es aquella que
tiene el Ags +, (hepatitis crnica activa, hepatitis aguda o portadora). Si
adems el Age es positivo, el riesgo asciende hasta un 90%. El momento
de mayor riesgo es el momento del parto.
La mayora de las veces, la enfermedad en el neonato es asintomtica,
pero tiene una alta probabilidad de evolucionar hacia una forma crnica
y sufrir degeneracin maligna.
Para evitar este curso es fundamental realizar prolaxis con gammaglo-bulina
especca en las 12 primeras horas de vida y primera dosis de vacuna. Si la
prolaxis se realiza correctamente, podr recibir lactancia materna. Posteriormente hay que seguir la pauta de vacunacin de VHB a los 2 y 6 meses.

y calcicaciones periventriculares (en el caso de la toxoplasmosis, stas


son difusas).
La infeccin congnita por CMV es la infeccin viral congnita ms frecuente (respuesta 1 falsa). El 90% de los recin nacidos estn asintomticos
(respuesta 2 correcta), pero si tienen clnica pueden presentar ictericia,
hepatoesplenomegalia, calcicaciones, hepatitis,etc. La neumonitis es
la forma ms caracterstica de infeccin postnatal.
La secuela ms frecuente es la hipoacusia neurosensorial, no de transmisin (respuesta 3 falsa).
La afectacin ocular tpica es la coriorretinitis (respuesta 4 falsa).
Estos nios suelen desarrollar microcefalia evolutivamente (respuesta
5 falsa).

Pregunta 26.- R: 2
El caso clnico nos presenta a un RN con clnica de CMV congnita. Los
datos claves que nos permiten llegar al diagnstico son: coriorretinitis

Pregunta 27.- R: 2
Recuerda que si la infeccin de la gestante por toxoplasma ocurre en el
primer trimestre, el RN puede presentar la ttrada de Sabin consistente

IgGCTOMEGALOVIRUS

TORMOPLAMA

Infeccin materna

Se reactiva asintomtica 1-2%


Se infecta sintomtica 1-2%0

Se infecta

Transmisin

Placentaria 1.er trimestre ms frecuente, ms grave


Canal de parto
Lactancia

Placentario
1.er trimestre ms grave
3.er trimestre ms frecuente

Ansioltico ms frecuente

5-18% secuelas tardas, hipoacusia neurosensorial bilateral


y severa.

Sin tratamiento coriorretinitis

Sntoma

Corriorretinitis
Microcefalia
Calcicaciones periventriculares

SABIN
Coriorretinitis
Hidrocefalia
Calcicaciones intracraneales difusas
Convulsiones

Screening prenatal

Prenatal Ac antiCMV (no se hace prevencin)

1. determinacin:
Negativo: vigilar
Positivo: hacer una 2. determinacin:
- Disminuido igual: infeccin pasada
- Aumantado: tratar

Diagnstico

Aislamiento y cultivo en orina inclusiones en ojo de bho


IgM
IgG o estable a las 6 semanas

Aislamiento en placenta (sangre?)


IgM
IgG o estable a las 6 semanas

Tratamiento

No
Ganciclovir a veces

Siempre:
Sintomtico: primetadina + sulfadicina 6 meses
primetadina + sulfamicina/espiromicina 6 meses
Asintomtico: espiromicina y valorar serologa

Pregunta 27. Diferencias entre toxoplasma y citomegalovirus.


CTO Medicina C/Francisco Silvela, 106 28002 - Madrid

Tfno. (0034) 91 782 43 30/33/34 E-mail: secretaria@ctomedicina.com www. ctomedicina.

Comentarios de Test a distancia 1. vuelta

Pediatra
en coriorretinitis), manifestacin ms habitual, aunque suele aparecer
de forma tarda calcicaciones intracraneales difusas, hidrocefalia y
convulsiones.
Si ocurre en el segundo o tercer trimestre, la infeccin fetal es ms
frecuente, pero el RN suele presentar sntomas leves o enfermedad
subclnica.

Pregunta 28.- R: 1
Estamos ante un nio RN CIR (edad gestacional de 38 semanas y peso
< 2.500 g) con clnica de erupcin petequial y hepatoesplenomegalia.
Esto lo puede presentar cualquier infeccin connatal.
A continuacin nos describen la clnica tpica del DAP: soplo continuo
en 2. espacio intercostal izquierdo con pulsos femorales saltones. Es un
caso de rubola congnita. Las lesiones seas tambin son las tpicas
de la rubola, las que nos encontraramos en la slis seran las mismas,
pero con reaccin peristica.

Al contrario que en todas las infecciones connatales, la rubola tiene


ms riesgo de trasmisin en el 1.er trimestre. Tambin es ms grave
si se infecta en este trimestre.
La trada de Gregg se caracteriza por:
- Hipoacusia neurosensorial: es la manifestacin ms habitual.
- Cardiopata: la ms frecuente, el DAP.
- Afectacin ocular: lo ms habitual son las cataratas.
Un RN infectado por rubola congnita puede eliminar el virus en
las secreciones hasta ao y medio despus del nacimiento, por lo
que es obligatorio aislarlo de mujeres embarazadas.

Pregunta 29.- R: 1
De los factores que se citan, el ms relacionado con el CIR tipo simtrico
(CIR tipo I) es la infeccin congnita.
Recuerda que hay que sospechar una infeccin connatal ante la presencia de CIR tipo simtrico, hepatoesplenomegalia, adenopatas, ictericia,
anemia y trombopenia en un recin nacido.
La causa ms frecuente de CIR tipo asimtrico (CIR tipo II) es la insuciencia placentaria.
Pregunta 30.- R: 4
Todo el caso clnico hace referencia a la infeccin por el virus herpes
simple, siendo el ms frecuente el tipo II (75-90%).
La clnica suele comenzar a la semana, con grave afectacin del estado
general, fontanela abombada y vesculas en la zona de presentacin (en
la prctica clnica slo aparecen en un 70% de los casos). Se asocia con
una elevada mortalidad. El neonato contrae la infeccin cuando pasa por
el canal del parto, por lo tanto, lesiones genitales activas en el momento
del parto contraindican un parto vaginal y es indicacin de cesrea.
El tratamiento en este nio sera la administracin de aciclovir intravenoso.
No te olvides de repasar las caractersticas de las infecciones connatales.

Pregunta 28. Manifestaciones clnicas de la rubola congnita.


De la rubola congnita para el MIR debemos recordar:
MOMENTO
DE ADQUISICIN

Pregunta 31.- R: 2
El granuloma, que aparece al caerse el cordn (por infeccin leve o por
epitelizacin incompleta), es un tejido blando, granular, vascular y rojizo
o rosado, que puede tener a veces una secrecin mucopurulenta.
Las masas umbilicales se pueden clasicar en grandes (hernia umbilical,

VA
DE ADQUISICIN

ESTIGMAS CARACTERSTICOS

Rubola

Sobre todo 1.er trimestre

Placentaria

Trada de Gregg: catarata, sordera/cardiopata (ductus sobre todo)


Otros: coriorretinitis, retinopata sal-pimienta, estenosis pulmonar
perifrica, prpura trombopnica

CMV
(la ms frecuente)

Ms frecuente en 3.er trimestre


Ms grave en 1.er trimestre: ms
clnica

Placentaria, canal,
leche

Calcicaciones periventriculares cerebrales, microcefalia, coriorrenitis


Muchos asintomticos (con o sin secuelas tardas: la ms frecuente es
la sordera)

VHS
(75-95% VHS II)

Ms frecuente en parto

Canal,
transplacentario,
postparto (90%)

Precoz: CIR, vesculas, alteraciones neurolgicas y oculares


(queratoconjuntivitis)
Tarda (en parto): sepsis, encefalitis con lesin del lbulo temporal,
alteracin ocular (sobre todo queratoconjuntivitis y vesculas cutneas)

Toxoplasmosis

Ms grave en 1.er trimestre, ms


frecuente en el 3.er trimestre

Placentaria

Calcicaciones cerebrales perifricas


Ttrada de Sabin: coriorretinitis, hidrocefalia, convulsiones,
calcicaciones

Varicela

Ms grave en 1.er trimestre. Si se


presenta en el 3.er trimestre, es ms
grave cuanto ms cerca del parto

Placentaria

Precoz: cicatrices, atroa de miembros, malformaciones oculares


y cerebrales
Tarda: vesculas, afectacin visceral y dicultad respiratoria

Slis

Sobre todo en 3.er trimestre

Placentaria

Precoz (< 2 aos): pngo siltico, hepatoesplenomegalia, rinitis (trada


de la slis precoz)
Tarda (> 2 aos): trada de Hutchinson (sordera + queratitis
+ alteraciones dentarias), articulacin de Clutton, periostitis

Pregunta 30. Infecciones connatales.


CTO Medicina C/Francisco Silvela, 106 28002 - Madrid

Tfno. (0034) 91 782 43 30/33/34 E-mail: secretaria@ctomedicina.com www. ctomedicina.com

Comentarios de Test a distancia 1. vuelta

Pediatra
onfalocele y gastrosquisis) y pequeas (granuloma, plipo y onfalitis).
Recuerda las principales caractersticas de cada uno de ellos.
Hernia umbilical

Paramedio
(lat. dcho. + frec.)

Peritoneo y piel

Peritoneo

Sin peritoneo

- Estrangulacin
(rara)
- Reduccin
espontnea

- Anomalas
cromosmicas
- Extroa vesical
- Sd. Beckwith
(macrosoma e
hipoglucemia)

Atresia intestinal

Quirrgico, si:
- Estrangulacin
- Crecimiento
progresivo
- Persiste a los
3-5 aos

Correccin
quirrgica precoz
(peor pronstico)

Correccin
quirrgica precoz
(buen
pronstico)

Umbilical

CUBIERTAS

TRATAMIENTO

Gastrosquisis

Umbilical

DEFECTO DE
CIERRE

COMPLICACIONES

Onfalocele

Su densidad es mayor que la leche humana madura (respuesta 3 correcta)


pues contiene mayor cantidad de protenas y minerales que sta pero no
hidratos de carbono (respuesta 2 falsa) ni grasa, y una serie de factores
inmunitarios importantes en la defensa del RN (respuesta 5 correcta).
Con los das, el calostro es sustituido por una leche de transicin que
se convierte en madura hacia la 3.-4. semana (respuesta 4 correcta).
Pregunta 36.- R: 2
Es importante recordar las principales caractersticas que las diferencian.
LECHE HUMANA
Caloras

670 kcal/l

670 kcal/l

Protenas

1-1,5 g
Casena 30%
Seroprotenas 70%

3-4,5 g
Casena 80%
Seroprotenas 20%

Hidratos de carbono

7 g lactosa y otras

4,5 g lactosa

Grasas

3,5 g % c. grasos
esenciales
c. grasos cadena larga
insaturados
Colesterol

3,5 g
Escasos c. grasos
esenciales
c. grasos
saturados

Minerales

+++ (5 veces)

Hierro

Cobre

++

Flor

Relacin calcio/fsforo

Vitamina A

++

Vitamina B

++

Vitamina C

Escasa

Vitamina D

Escasa

Vitamina E

++

Vitamina K

Escasa

Nitrgeno
no proteico

+++

Pregunta 31. Patologa umbilical.


Sobre secreciones por el ombligo han preguntando en el MIR:
Persistencia del uraco: se segrega por el ombligo un lquido amarillento similar a la orina con un pH cido.
Persistencia del conducto onfalomesentrico (fstula intestinal):
aparece un lquido amarillento con un pH alcalino.
Pregunta 32.- R: 4
El hipotiroidismo congnito es una patologa relativamente comn que
se debe descartar en todos los neonatos. Su causa ms frecuente es la
disgenesia tiroidea (respuesta 1 correcta).
En el recin nacido los signos y sntomas de hipotiroidismo son muy
sutiles y la clnica se va instaurando de forma progresiva (respuesta
4 falsa). Aparece facies peculiar, estreimiento, ictericia prolongada,
letargia, hernia umbilical, fontanelas amplias, retraso en la maduracin sea y puede asociarse una sordera neurosensorial (respuesta
2 correcta).
La determinacin de TSH en sangre, obtenida entre los 2 y los 5 das de
vida, es la prueba de cribado rutinaria. Cuando se conrma el diagnstico
es necesario iniciar el tratamiento precoz con levotiroxina para evitar la
aparicin de retraso mental irreversible (respuesta 3 correcta).
Pregunta 33.- R: 4
Recordar los siguientes datos de forma muy simplicada:
1,5 m: sonrisa social.
3 m: inicio sostn ceflico.
6 m: inicia sedestacin.
9 m: movimiento de pinza.
10-12 m. Inicia bipedestacin.
12-15 m: primeros pasos y emite bislabos.
24 m: sube/baja escaleras, corre y apila 4 o 6 cubos para formar una
torre.
5-10 aos: comprende que la muerte es un fenmeno permanente.

LECHE DE VACA

Pregunta 36. Comparacin entre leche humana


y leche de vaca.
Pregunta 37.- R: 3
La causa ms frecuente de talla baja en el nio son las dos variantes de
la normalidad.
TALLA
BAJA FAMILIAR

RETRASO CONSTITUCIONAL
DEL CRECIMIENTO

Talla RN

Menor de la
normal

Normal

Velocidad
de crecimiento

Normal

Menor de lo normal, luego normal

Pregunta 34.- R: 5
Recuerda que el peso del nio se duplica al 5. mes, se triplica al ao y
se cuadriplica a los 2 aos. La talla del nacimiento se duplica alrededor
de los 4 aos de edad (respuesta 5 falsa).

Antecedentes
familiares

De talla baja

De pubertad retrasada

Edad sea

Igual a edad
cronolgica

Igual a edad talla


Retrasada respecto a la edad
cronolgica

Pregunta 35.- R: 2
Del calostro debes saber que es la leche de los 2-4 primeros das despus
del parto (respuesta 1 correcta).

Talla nal

Disminuida

Normal o algo disminuida

CTO Medicina C/Francisco Silvela, 106 28002 - Madrid

Pregunta 37. Tipos de talla baja no patolgica.

Tfno. (0034) 91 782 43 30/33/34 E-mail: secretaria@ctomedicina.com www. ctomedicina.

Comentarios de Test a distancia 1. vuelta

Pediatra
Pregunta 38.- R: 5
Si tenemos presente la tabla del comentario de la pregunta anterior
podemos deducir fcilmente que estamos ante un caso de talla baja
familiar. La curva de crecimiento es inferior al percentil 3, no hay discordancia entre la edad sea del nio y su edad cronolgica y, adems,
existe historia familiar de talla baja. Las pruebas de laboratorio sern
normales y la talla adulta nal previsible ser baja, pero dentro de los
lmites esperados para su talla gentica.

seguridad (UCI, quirfano) y administracin de antibiticos, generalmente cefalosporinas de 3. generacin. Los corticoides pueden ser
beneciosos en la primera parte del tratamiento pero la adrenalina no
es til (respuesta 3 falsa). Con el tratamiento adecuado, el cuadro suele
remitir en 24-48 horas.
En los casos dudosos nunca hay que intentar visualizar la epiglotis,
sino realizar una Rx lateral de faringe donde se apreciar una epiglotis
engrosada.

Pregunta 39.- R: 3
Es un cuadro de laringitis aguda, obstruccin de la va area superior, de
etiologa viral (virus parainuenzae en el 75% de los casos) caracterizado
por un antecedente previo de infeccin de vas altas con ebre de 38-38,5
C, tos perruna o metlica, afona, estridor de predominio inspiratorio y
dicultad respiratoria variable con empeoramiento nocturno.
Afecta a nios entre 3 meses y 5 aos y tiene predominio estacional,
producindose la mayora de las veces en otoo.
El tratamiento consiste en humedad ambiental, corticoides (en aerosol
o sistmicos) y adrenalina en aerosol.

Pregunta 42.- R: 5
Estamos ante un cuadro de bronquiolitis aguda que se dene como
primer episodio de dicultad respiratoria con sibilancias. El agente ms
frecuente es el VRS. Es una enfermedad estacional de los meses de invierno y primavera.
Afecta a nios menores de 2 aos, principalmente lactantes. Comienza por
un cuadro catarral, con tos, mocos y febrcula, y en los das siguientes evoluciona hacia un cuadro de obstruccin de vas areas distales con sibilancias
(dato ms tpico de la exploracin), espiracin alargada, roncus dispersos e
hipoventilacin. Esta fase crtica suele durar 2-3 das con posterior mejora
(respuestas 1 y 3 correctas). En lactante muy pequeo pueden producirse pausas de apnea con cianosis como nica manifestacin (respuesta 2 correcta).
El tratamiento consiste en medidas generales (oxigenoterapia, monitorizacin, etc.), beta-2 agonistas inhalados y adrenalina nebulizada. Ni los
corticoides ni los antibiticos estn indicados.
La ribavirina inhalada estara recomendada en bronquiolitis graves
que afectan a nios de riesgo (patologa pulmonar o cardaca de base).
Actualmente se intenta prevenir su aparicin con la administracin de
palivizumab, anticuerpo monoclonal anti-VRS indicado en prematuros,
cardipatas, neumpatas e inmunodeprimidos.
La secuela a largo plazo ms frecuente es la hiperreactividad bronquial.
La bronconeumona y OMA son poco frecuentes (respuesta 5 falsa).

Pregunta 40.- R: 2
Es un cuadro clnico tpico de laringitis pero sin ebre y sin ningn antecedente de infeccin de vas altas de caractersticas virales. Se trata,
por tanto, de una laringitis espasmdica o estridulosa.
Es el momento de repasar las caractersticas de los tipos de crup.
Pregunta 41.- R: 3
La epiglotitis es una enfermedad tpica de los nios menores de 5 aos.
En la actualidad, los principales responsables de esta entidad son los
cocos grampositivos como el S. pyogenes, S. pneumoniae y S. aureus. Por
detrs de stos, hay que tener en cuenta tambin a su agente clsico,
H. inuenzae tipo B (cada vez menos frecuente gracias a la vacunacin
universal).
La clnica suele cursar de forma brusca por la noche, con ebre alta y
aspecto sptico, babeo e intensa dicultad respiratoria con estridor
inspiratorio. Empeora con el llanto y con el decbito supino, haciendo
que el nio permanezca en posicin de trpode. No suele existir tos ni
afona y el estridor no es tan ruidoso como el de las laringitis.
En el tratamiento, lo prioritario es asegurarse la va area mediante
intubacin (a ser posible nasotraqueal), siempre en condiciones de

LARINGITIS AGUDA
(estridulosa)

Pregunta 43.- R: 4
El cuadro que presenta este nio es compatible con una brosis qustica.
Las manifestaciones clnicas suelen ser:
Respiratorias: bronquiolitis recurrentes, tos crnica, neumona, plipos
nasales, sinusitis, etc.
Digestivas: leo meconial, esteatorrea, malabsorcin, prolapso rectal,
cirrosis biliar, etc.
Otras: deshidratacin, malnutricin, diabetes, azoospermia.

LARINGOTRAQUEITIS VIRAL

EPIGLOTITIS AGUDA

TRAQUETIS
BACTERIANA

Etiologa

Alergia + psicolgico

Virus parainuenzae 1
(el ms frecuente)

Antecedentes

No hay

Catarro vas altas


(paciente y/o familiar)

Clnica

Espasmo larngeo recortado


(generalmente nocturno)
No ebre

Fiebre, tos, disnea alta, estridor


inspiratorio

Fiebre alta + babeo + disfagia +


disnea + cabeza extendida

Fiebre + estridor mixto

Duracin

1-2 noches

Das-semanas

Fulminante

Das-semanas

Tratamiento

Ambiente tranquilo, humidicar


Corticoides
Adrenalina racmica en aerosol

Intubar + oxgeno
Antibitico
Tto. en UVI

Oxgeno
Intubacin si es
preciso
Cloxacilina i.v.

Cocos grampositivos

S. aureus

Crup vrico

Pregunta 40. Tipos de crup.


CTO Medicina C/Francisco Silvela, 106 28002 - Madrid

Tfno. (0034) 91 782 43 30/33/34 E-mail: secretaria@ctomedicina.com www. ctomedicina.com

Comentarios de Test a distancia 1. vuelta

Pediatra
Para el diagnstico se requiere manifestaciones clnicas compatibles y/o
antecedente de hermano/a con enfermedad conrmada y/o cribado
neonatal positivo ms un criterio de conrmacin (test del sudor positivo
o presencia de mutaciones en el estudio gentico o test de diferencia
de potencial nasal positivo).
Para el tratamiento de estos pacientes es fundamental el cuidado del
pulmn. Se debe realizar diariamente sioterapia respiratoria y tratamiento agresivo de las infecciones pulmonares. Adems, se llevar a
cabo un adecuado soporte nutricional y tratamiento especco de las
posibles complicaciones.
Pregunta 44.- R: 1
La traquetis bacteriana tiene como principal responsable a S. aureus
(respuesta 1 falsa). Hay que sospechar esta entidad cuando, tras un
cuadro de crup vrico, se produce un empeoramiento progresivo con
ebre, dicultad respiratoria de intensidad creciente y estridor mixto.
Es una entidad grave que suele requerir la hospitalizacin del nio para
instaurar tratamiento antibitico (cloxacilina i.v.) y oxigenoterapia a
demanda. Si aparece gran dicultad respiratoria puede ser necesaria
la intubacin.
Pregunta 45.- R: 4
En la patogenia de la brosis qustica est implicada una alteracin de
la regulacin de los canales inicos de las membranas celulares, lo que
produce secreciones deshidratadas y espesas. A nivel pulmonar, la lesin anatomopatolgica inicial es la bronquiolitis (respuesta 1 correcta).
El leo meconial supone la forma de debut neonatal en aproximadamente
un 10% de los casos (respuesta 2 correcta). La funcin endocrina del
pncreas se afecta con el tiempo, pudiendo aparecer diabetes mellitus a
partir de los 10 aos de evolucin de la enfermedad (respuesta 3 correcta).
Debido a la prdida excesiva de sal, se ven cuadros de deshidratacin
con hiponatremia e hipocloremia, coincidiendo con gastroenteritis o en
pocas de calor (respuesta 5 correcta).
La aspergilosis broncopulmonar alrgica es relativamente frecuente en
estos pacientes. Hay que sospecharla ante la presencia de esputo herrumbroso, aislamiento de Aspergillus fumigatus o la presencia de eosinlos
en una muestra fresca de esputo. El tratamiento ser corticoterapia oral,
en los casos refractarios puede ser necesario el empleo de anfotericina
B en aerosol o de 5-uorocitosina sistmica (respuesta 4 falsa).
Pregunta 46.- R: 4
Es necesario iniciar el tratamiento de la brosis qustica antes de que
la afectacin pulmonar sea importante. Se ha visto que el empleo de
sioterapia respiratoria alarga la supervivencia de estos pacientes.
Es til el empleo moderado de enzimas pancreticas para disminuir la
esteatorrea, as como el aporte de suplementos de vitaminas liposolubles a la dieta.
En los pacientes con brosis qustica, la neumona por sobreinfeccin
bacteriana se debe principalmente a Pseudomonas aeruginosa, variedad
mucoide. Este patgeno puede colonizar la va respiratoria dando lugar
a un estado de portador crnico difcil de revertir (respuesta 4 falsa).
Pregunta 47.- R: 4
La causa ms frecuente de GEA en nuestro medio es el rotavirus. Es una
enfermedad tpica de lactantes que produce una diarrea lquida, SIN
sangre. Es proceso autolimitado, que cede en 3-10 das (respuesta 4
falsa). El diagnstico se basa en la realizacin de un test de ELISA en las
heces para detectar el Ag.
El tratamiento es sintomtico. Existen dos tipos vacunas atenuadas
de 2 o 3 dosis respectivamente de reciente aparicin y administracin oral.
CTO Medicina C/Francisco Silvela, 106 28002 - Madrid

Pregunta 48.- R: 1
Estamos ante un caso de alergia a protenas de leche de vaca. El cuadro
tpico consiste en inicio de la clnica tras la ingesta de un bibern de
frmula adaptada de leche de vaca. La protena ms frecuentemente
implicada es la betalactoglobulina (respuesta 2 correcta). La sintomatologa ms habitual es la cutnea (urticaria), seguida de la digestiva
(vmitos, diarrea) y en ocasiones respiratoria (bronco espasmo).
El cuadro est mediado por un mecanismo inmunolgico tipo IgE.
Afecta a individuos atpicos o con antecedentes familiares de atopia
(respuesta 1 falsa).
El diagnstico es fundamentalmente clnico, se basa en la exclusin
de dichas protenas y consecuente desaparicin de los sntomas y la
posterior provocacin (respuesta 4 correcta). Se emplean las pruebas
cutneas, que sern positivas (respuesta 3 correcta).
El tratamiento consiste en la suspensin de la frmula adaptada, reemplazndola por frmulas especiales (hidrolizado de protenas). Suele ser
un proceso autolimitado (respuesta 5 correcta), la mayora de los nios
pueden tomar leche de vaca a partir de los 2 aos.
Recuerda las diferencias entre INTOLERANCIA y ALERGIA a las protenas
de leche de vaca.
APLV

IPLV

Patogenia

Mediado IgE

No IgE

Sntomas

Digestivos y alrgicos

Digestivos

Clnico
Provocacin
Diagnstico

InmunoCAP
Prick
HIDROLIZADOS DE PLV

Tratamiento

AntiH1
Adrenalina s.c.

Pregunta 48. Intolerancia-alergia a protenas de leche de vaca.


Pregunta 49.- R: 4
Caso clnico de atresia con fstula traqueoesofgica. Segn la clasicacin de Ladd, el tipo ms frecuente es la atresia proximal con fstula
distal (tipo III).
El diagnstico se realiza por la sospecha clnica (salivacin excesiva,
cianosis y atragantamiento con las tomas, etc.) e imposibilidad para
pasar una sonda nasogstrica.
El tratamiento es quirrgico, siendo el reujo gastroesofgico la complicacin postquirrgica ms frecuente, que suele ser grave (respuesta
4 correcta). Otras complicaciones menos habituales son la fstula de
la anastomosis, la recidiva de la fstula traqueoesofgica, la estenosis
esofgica y la traqueomalacia.
Pregunta 50.- R: 5
Estamos ante un caso de RGE siolgico, caracterizado por pequeos
vmitos sin fuerza despus de las tomas, por lo dems, el nio se encuentra asintomtico con peso y talla en percentiles normales.
El RGE es muy frecuente en el primer ao de vida, hasta un 85%. Generalmente desaparece hacia los 2 aos.
En un 10% de los casos, se habla de RGE patolgico o enfermedad por
reujo gastroesofgico (ERGE) cuando se asocia con escasa ganancia
ponderal, esofagitis y clnica respiratoria.
El diagnstico se basa en la historia clnica. Slo en el RGE patolgico est
indicada la realizacin de pruebas complementarias: trnsito superior
con bario (primera prueba a realizar) y una pHmetra de 24 horas (para
cuanticar el grado de reujo es la prueba ms sensible y especca, se

Tfno. (0034) 91 782 43 30/33/34 E-mail: secretaria@ctomedicina.com www. ctomedicina.

Comentarios de Test a distancia 1. vuelta

Pediatra
REFLUJO GASTROESOFGICO

ESTENOSIS HIPERTRFICA
DE PLORO

Momento de aparicin

1. semana

2. a 3. semana

Vmitos

Alimentarios sin fuerza

Alimentarios a chorro

Otros sntomas

Sntomas respiratorios
Disminucin del crecimiento

Masa palpable
Alcalosis hipoclormica

Diagnstico

Tratamiento

Clnica + respuesta a tratamiento

pHmetra 24 h

endoscopia
Medidas posturales
Cisaprida
Ciruga (funduplicatura de Nissen)

ATRESIA PILRICA

Alimentarios

ATRESIA DUODENAL

Biliosos
Abdomen excavado
Asociar al Down

Evolucin
Rx: signo de la cuerda
1 burbuja gstrica

RX:
1 burbuja gstrica
Ausencia de gas distal

RX:
1 burbuja gstrica
1 burbuja duodenal

Ciruga:
Piloromiotoma
Fredet-Ramsted

Ciruga

Ciruga

Pregunta 51. Vmitos del lactante.

realiza cuando existen manifestaciones extradigestivas del reujo). El


tratamiento en los casos leves consiste en medidas posturales y espesantes de las tomas. En casos de reujos ms graves se administraran
estimulantes del peristaltismo (cisaprida, domperidona) o tratamiento
quirrgico (funduplicatura de Nissen).
Pregunta 51.- R: 5
La clnica tpica de un cuadro de estenosis hipertrca de ploro consiste
en vmitos proyectivos inmediatos tras las tomas, alimenticios, no biliosos,
tras el vmito el nio se queda irritable y hambriento. Comienza entre
las 3-6 semanas de vida, siendo el momento ms habitual en torno a los
20 das de vida (respuesta 1 falsa). Es ms habitual en varones de raza
blanca (respuesta 2 falsa).
Debido a las prdidas de hidrogeniones y cloruros con los vmitos se
produce una alcalosis metablica hipoclormica (respuesta 3 falsa) con
normopotasemia o hipopotasemia.
La prueba diagnstica de eleccin ante la sospecha de este cuadro sera
una ecografa abdominal (respuesta 4 falsa). El hallazgo frecuente en la
radiografa simple de abdomen es la distensin gstrica con escaso gas
distal. El trnsito digestivo est en desuso en la actualidad, se reserva
para pacientes en los que la ecografa no es concluyente, donde aparece
de forma tpica la imagen del signo de la cuerda.
El tratamiento es quirrgico: pilorotoma extramucosa de Ramsted (respuesta 5 correcta). Recuerda que pueden existir vmitos postoperatorios
secundarios al edema de ploro producidos por la propia incisin.
Repasa los cuadros clnicos que producen vmitos en el lactante.
Pregunta 52.- R: 4
Caso tpico de atresia duodenal con vmitos biliosos en las primeras 2448 horas, tras las primeras tomas, con abdomen excavado por ausencia
de aire distal. Importante el antecedente de sndrome de Down, ms
comn en estos nios. Con mayor frecuencia la atresia se localiza a nivel
de la 3. porcin del duodeno. En la Rx de abdomen es tpico encontrar
la imagen de doble burbuja. Recuerda los datos clnicos caractersticos:
Estenosis hipertrca de ploro: vmitos no biliosos a las 2-3 semanas
de vida.
Enfermedad de Hirschprung: estreimiento y en ocasiones vmitos
fecaloideos.
Divertculo de Meckel: sangrado indoloro rectal sin vmitos.
Invaginacin intestinal: episodios de dolor intenso, siendo muy raro
en el periodo neonatal.
Pregunta 53.- R: 4
El divertculo de Meckel es un resto del conducto onfalomesentrico.
Es la malformacin digestiva ms frecuente, apareciendo en un 2% de
CTO Medicina C/Francisco Silvela, 106 28002 - Madrid

lactantes (respuesta 1 correcta). Se localiza a unos 50-70 cm de la vlvula


ileocecal (respuesta 2 correcta).
La clnica ms frecuente suele consistir en una hemorragia rectal INDOLORA (respuesta 4 falsa) e intermitente debido a la ulceracin de la
mucosa ileal adyacente al divertculo con mucosa ectpica (gstrica o
pancretica).
La tcnica diagnstica ms sensible es la gammagrafa con Tc-99.
El tratamiento es quirrgico.
Pregunta 54.- R: 3
Cuadro tpico de invaginacin intestinal. Recuerda que es la causa ms
frecuente de obstruccin intestinal entre los 3 meses y los 6 aos. Nio
que presenta episodios repetidos de llanto y encogimiento de piernas
con posterior decaimiento. Si la clnica evoluciona hasta un 60%, puede
presentar deposicionesen jalea de grosella, heces con sangre roja fresca
y moco. La forma ms frecuente es la ileoclica.
Para el diagnstico inicial se suele realizar una ecografa abdominal.
Una invaginacin intestinal es siempre una urgencia. Si han transcurrido
menos de 48 horas y no hay signos de perforacin, se realiza tratamiento con enemas de bario o con aire, para intentar la reduccin. Si hay
perforacin intestinal o distensin abdominal de ms de 48 horas de
evolucin, se preere tratamiento quirrgico.
La tasa de recurrencia es mayor en los pacientes en los que se realiza
reduccin hidrosttica (10-20%) que en los tratados quirrgicamente
(3%).
Pregunta 55.- R: 4
La enfermedad de Hirschprung o megacolon congnito se caracteriza
por estreimiento crnico desde el nacimiento, asociado a un estancamiento o retraso ponderal (por malabsorcin) secundario a un trastorno
en la migracin de los neuroblastos que supone la ausencia de clulas
ganglionares y plexos mesentricos.
Para el diagnstico es til el tacto rectal, donde se aprecia una ampolla rectal vaca de heces (respuesta 3 correcta). En la manometra
se observa un aumento paradjico del tono del esfnter anal interno
(respuesta 4 falsa) ante un incremento de presin a ese nivel. En el
enema opaco se objetiva un retraso en la eliminacin de contraste
(respuesta 5 correcta). El diagnstico denitivo lo dar la biopsia: en el
segmento afectado se observa ausencia de clulas ganglionares con
incremento de la acetilcolinesterasa y aumento de las terminaciones
nerviosas (respuesta 1 correcta).
El tratamiento es quirrgico, con la reseccin de todo el segmento
aganglinico. En el RN con obstruccin intestinal se realiza colostoma
y descenso con posterior cierre de colostoma. Si no hay obstruccin,
se realiza un descenso primario precoz.

Tfno. (0034) 91 782 43 30/33/34 E-mail: secretaria@ctomedicina.com www. ctomedicina.com

10

Comentarios de Test a distancia 1. vuelta

Pediatra
Pregunta 56.- R: 5
Los factores que intervienen en la patogenia de la celiaqua son:
HLA: DR3, DR4, DR7 , DQ y W2.
Factores inmunolgicos: los linfocitos de la lmina propia se inmunizan frente a la gliadina.
Factores ambientales.
La clnica se caracteriza por un cuadro de malabsorcin, de inicio ms
frecuente entre los 6 meses y los 2 aos. Suelen presentar esteatorrea,
estancamiento de la curva ponderal, disminucin de la masa muscular,
irritabilidad y distensin abdominal importante.
El diagnstico se basa en los test serolgicos, Acs antigliadina, antirreticulina, antiendomisio y antitransglutaminasa, estos ltimos los ms
sensibles y especcos de todos, y en la realizacin de biopsia intestinal,
necesaria para establecer diagnstico. La biopsia se har, al menos, en
una ocasin, estando el paciente consumiendo gluten y se tomarn
varias muestras de porcin duodenal o inicio de yeyuno. Clsicamente
se realizaban tres biopsias: 1. (sin exclusin), demuestra atroa; 2. (tras
exclusin), demuestra recuperacin de mucosa; y 3. (tras prueba de
provocacin), demuestra atroa. Actualmente la 2. y la 3. se realizan
nicamente cuando:
El inicio de la clnica se produjo por debajo de los 2 aos de edad
(riesgo de falsos positivos en la biopsia).
La respuesta clnica a la exclusin del gluten de la dieta no ha sido
concluyente y existe duda diagnstica.
La retirada del gluten se hizo sin realizacin de primera biopsia.
Para llevar a cabo la 2. biopsia, se debe esperar un mnimo de 24 meses
tras retirada del gluten y no ha de practicarse hasta los 6 aos de vida.
El tratamiento consiste en la exclusin de por vida del trigo, cebada, centeno y +/- avena y triticale. Pueden tomar libremente maz, soja y arroz.

Se har, al menos en una ocasin, estando el paciente consumiendo


gluten (respuesta 4 falsa). Una vez asegurado el diagnstico se iniciar
el tratamiento a base de una dieta rigurosamente exenta de gluten de
por vida.
La biopsia intestinal en el caso de la enfermedad celaca no es patognomnica (respuesta 3 falsa), pero s muy caracterstica: aparece un
inltrado inamatorio en la lmina propia, junto con hiperplasia de las
criptas y atroa vellositaria.
El seguimiento se realizar valorando la evolucin clnica y determinando
peridicamente los anticuerpos para evaluar el cumplimiento de la dieta
(respuesta 5 falsa). Los anticuerpos ms sensibles y especcos son los
IgA antitransglutaminasa (respuesta 2 correcta).
Pregunta 58.- R: 2
Estamos ante un cuadro de atresia de vas biliares extrahepticas (AVBEH);
lo tpico es RN a trmino sano y con fenotipo normal que desarrolla
cuadro de colestasis (predominio de bilirrubina directa, coluria, acolia
o hipocolia) a las 2-3 semanas de vida. Se puede asociar a poliesplenia,
malrotacin intestinal, anomalas vasculares, etc. La aparicin de varios
casos en la misma familia es rara.
La prueba diagnstica sera la realizacin de un HIDA, donde se aprecia
captacin del contraste por el hgado, pero que no llega al intestino (ausencia de excrecin). El diagnstico de certeza se consigue mediante laparotoma exploradora con realizacin intraoperatoria de colangiografa.
Su tratamiento denitivo es el trasplante heptico (el 80% de pacientes
lo requerir a lo largo de su evolucin). Previamente se intentar realizar
una hepatoportoenterostoma o tcnica de Kasai, para intentar restablecer el ujo biliar y disminuir el dao (respuesta 2 falsa). Actualmente, la
AVBEH constituye la primera indicacin del trasplante heptico infantil.

Pregunta 58. Colestasis neonatal.

Pregunta 56. Diagnstico de la enfermedad celaca.


Pregunta 57.- R: 2
Pregunta importante sobre el diagnstico de la enfermedad celaca. Ante
la presencia de una alta sospecha clnica es necesaria la realizacin de
una biopsia intestinal para conrmar el diagnstico (respuesta 1 falsa).
CTO Medicina C/Francisco Silvela, 106 28002 - Madrid

Pregunta 59.- R: 3
Es un cuadro de malabsorcin de hidratos de carbono. Tiene como antecedente una GEA que lesiona el borde del enterocito, produciendo un
dcit transitorio de lactasa. La clnica se caracteriza por diarrea, dolor
clico, heces espumosas y cidas (pH < 5,5) que excorian la zona del
paal. Ante la sospecha se debe realizar un Clinitest en las heces que
ser positivo. El tratamiento consiste en la exclusin de la lactosa de la
dieta, que la mayora de las veces es transitoria.

Tfno. (0034) 91 782 43 30/33/34 E-mail: secretaria@ctomedicina.com www. ctomedicina.

11

Comentarios de Test a distancia 1. vuelta

Pediatra

El dcit de sacarasa-isomaltasa: produce la misma clnica pero el


Clinitest es negativo.
El dcit aislado de isomaltasa: es asintomtico.
El dcit de enteroquinasa: produce una malabsorcin de protenas
con clnica de desnutricin y edemas.
Gastroenteritis por rotavirus: la clnica es vmitos y diarrea lquida,
no cida.

Pregunta 60.- R: 2
La criptorquidia (descenso testicular incompleto) es el trastorno de la
diferenciacin sexual ms frecuente en los varones. Se estima que un
4,5 % de los nios tienen criptorquidia al nacimiento. Como el descenso
testicular se produce en la ltima fase de la gestacin, es ms frecuente
en prematuros. La mayora de los testculos no descendidos bajan al
escroto espontneamente en los primeros 3 meses de vida. Si sto no
ha ocurrido a los 6 meses de edad, es poco probable que lo haga ms
adelante. Las consecuencias de la criptorquidia permanente son:
Riesgo de esterilidad en los casos bilaterales.
Riesgo de degeneracin maligna a seminoma.
Hernias asociadas.
Torsin testicular en el lado afecto.
Efectos psicolgicos.
La edad idnea para realizar el tratamiento es entre los 9 y los 15 meses
de edad, preferentemente antes de los 2 aos. Consiste en el descenso y
la jacin quirrgica del testculo al escroto (orquidopexia); en todo caso
es primordial realizarla antes de la pubertad, sobre todo para eliminar el
riesgo de degeneracin maligna. El tratamiento con HCG es ms ecaz
cuanto mayor es la edad del nio y con una localizacin baja del teste.
Pregunta 61.- R: 4
En un lactante que presenta un cuadro clnico inespecco (vmitos,
febrcula y prdida de apetito), junto con leucocitosis, leucocituria y
nitritos positivos en orina, debemos sospechar una infeccin del tracto
urinario. Recuerda que la tira reactiva de orina es un mtodo sensible
que permite seleccionar aquellos nios a los que hay que realizar un
cultivo de orina, en este caso obtenido mediante puncin suprapbica,
e iniciar el tratamiento antibitico.
Pregunta 62.- R: 3
Respecto al RVU, hay que saber que es la anomala congnita ms frecuente de la unin ureterovesical. Se suele diagnosticar al estudiar las
ITU porque es causa predisponente de la misma. El paso retrgrado de
orina desde la vejiga al urter y pelvis renal desencadena una reaccin
inamatoria que puede dar lugar a la formacin de cicatrices renales y
si stas son extensas, se afectar la funcin renal (nefropata por reujo,
que es la causa de hasta un 20% de las insuciencias renales y la primera
causa de HTA en la infancia).
Para el diagnstico, la tcnica de eleccin es la cistografa miccional que
permite clasicar el reujo segn intensidad, grado de dilatacin ureteral
y deformidad calicial (grados I-V). Se debe realizar tambin una ecografa
renal para descartar anomalas estructurales y una gammagrafa renal,
tcnica de referencia para el diagnstico de cicatrices renales.
El reujo grado I y II en el 80% de los casos desaparece de forma espontnea al madurar el nio y no precisan tratamiento. Los objetivos del
tratamiento son prevenir la pielonefritis y la lesin renal.
La prolaxis antibitica a dosis bsica es el pilar fundamental del
reujo leve hasta que ste desaparezca.
En los casos que por su grado y repercusiones morfolgicas a nivel
renal, se espera que no desaparezca y/o lesione ms el rin, se
obtar por la ciruga, que puede ser endoscpica o abierta.
CTO Medicina C/Francisco Silvela, 106 28002 - Madrid

Pregunta 63.- R: 4
Caso clnico tpico de sndrome hemoltico urmico (SHU); nio
menor de 4 aos con antecedente de gastroenteritis enteroinvasiva
que presenta signos y sntomas de insuciencia renal aguda, clnica
neurolgica y afectacin gastrointestinal. El agente ms frecuente es E.
coli O-156). El patgeno produce unas toxinas que favorecen la lesin
a nivel endotelial capilar y arteriolar con la formacin de trombos de
plaquetas con trombopenia de consumo y anemia microangioptica
(respuesta 2 correcta). El rgano ms afectado en este cuadro es el
rin y es la causa ms frecuente de IRA en menores de 5 aos (respuesta 1 correcta).
El diagnstico se basa en:
Anemia hemoltica microangioptica: haptoglobina descendida con
esquistocitos en el frotis de sangre perifrica.
Plaquetopenia.
IRA: por ecografa se debe diferenciar de la trombosis venosa bilateral
(respuesta 3 correcta).
Se realizar tratamiento conservador de la IRA y alteraciones hidroelectrolticas y si no consiguen controlarse, estar indicada la dilisis
peritoneal (respuesta 4 falsa).
La funcin renal se recupera en el 90% de pacientes y las recidivas son
poco frecuentes.
Pregunta 64.- R: 2
Estamos ante un cuadro de torsin testicular. Se debe realizar diagnstico
diferencial con la orquioepididimitis.
La clnica consiste en un dolor agudo intenso, con tumefaccin escrotal, sin
ebre ni traumatismo previo. El reejo cremastrico suele estar anulado.
El diagnstico se basa fundamentalmente en la clnica, en caso de duda
se puede recurrir a la gammagrafa de ujo testicular o a la realizacin
de una eco-Doppler.
El tratamiento consiste en la reduccin manual o jacin quirrgica del
teste afecto y del contralateral.
El tiempo de viabilidad del testculo torsionado es de 4-6 horas, por lo
que siempre se debe considerar una urgencia quirrgica.
Recuerda las causas de escroto agudo.

Dolor

Estado general

Reejo
cremastrico

Inamacin

Intenso
Brusco
Continuo

Afectacin
importante
Cortejo
vegetativo

Abolido
Teste indurado
Teste
y edematoso
ascendido Eritema
o
escrotal
transverso

Mnima
afectacin

Presente

TORSIN
DEL
HIDTIDE

Menos
intenso
Gradual
Dolor a la
palpacin
del polo
superior

Menos
intensa
Punto azul
en polo
superior

ORQUIOEPIDIDIMITIS

Menos
intenso
Progresivo

Sndrome
miccional
asociado

Presente

Moderada
Edema
escrotal

TORSIN
TESTICULAR

Pregunta 64. Diagnstico diferencial del escroto agudo.


Pregunta 65.- R: 3
Caso clnico de PTI (prpura trombtica idioptica). Tpicamente aparece como un cuadro de petequias generalizadas, con disminucin
del nmero de plaquetas, tras un catarro de vas altas. No suele existir

Tfno. (0034) 91 782 43 30/33/34 E-mail: secretaria@ctomedicina.com www. ctomedicina.com

12

Comentarios de Test a distancia 1. vuelta

Pediatra
anemia ni otros signos de ditesis hemorrgica. El pronstico es bueno,
evolucionando hacia la recuperacin espontnea en la mayora de los
casos (respuesta 3 cierta).
Pregunta 66.- R: 4
Es importante que recuerdes:
Cncer infantil ms frecuente: leucemias (LAL tipo B). Los linfomas
siguen a las leucemias en frecuencia, salvando el grupo de edad de
entre 10 y 14 aos, en que superan a las anteriores.
Cncer slido infantil ms frecuente: tumores del SNC (astrocitoma).
Tumor slido extracraneal ms frecuente: neuroblastoma. El neuroblastoma, es ms habitual en varones y en la raza blanca, y el 90%
se diagnostican por debajo de los 5 aos de edad.
Tumor abdominal ms frecuente: neuroblastoma.
Masa abdominal ms habitual en el RN: hidronefrosis.
En cuanto a los tumores seos, en nios mayores y adolescentes,
predomina el osteosarcoma, seguido del sarcoma Ewing, que es el
ms frecuente en menores de10 aos.
Pregunta 67.- R: 3
El cuadro que se expone es una nia con un neuroblastoma. La edad de
aparicin tpica es en menores de 2 aos. Se suele localizar en el abdomen a nivel de las glndulas suprarrenales y la clnica ms frecuente es la
masa abdominal que cruza lnea media. Se asocia a hematoma palpebral
y hepatomegalia y entre los sndromes paraneoplsicos (no modican
el pronstico) gura el opsoclonus-mioclonus (caso clnico) y la diarrea
secretora. Para el diagnstico se utiliza la TC craneal, catecolaminas en
orina aumentadas y la gammagrafa con MIBG.
El tratamiento depende del estadio, realizndose ciruga, quimioterapia
y radioterapia. La supervivencia es del 50%. Recuerda las principales
diferencias entre neuroblastoma y nefroblastoma.
NEUROBLASTOMA

WILMS (NEFROBLASTOMA)

Epidemiologa

Tumor slido
extracraneal ms
frecuente en nios
Sobre todo < 2 aos
Delecin cromosoma 1

2. tumor abdominal ms
frecuente en nios
Mayores que los nios del
neuroblastoma ( 3 aos)
Delecin cromosoma 11

Localizacin

70% abdomen
(suprarrenal
o paramedial)
Pasa lnea media
20% trax

Rin
No pasa lnea media
A veces bilateral (familiares)

Clnica

Masa
Sd. paraneoplsicos
(VIP, opsoclonomioclono)

Asocia a veces
hemihipertroa, aniridia
(sd. de BeckwithWiedemann) y
malformaciones
genitourinarias
Masa abdominal
HTA

Tratamiento

Ciruga en estadio I
Otros: Qx + QT + RT

Bueno. A veces se diferencia


o regresa espontneamente
Metstasis a hgado, m.o.,
piel y hueso

Pronstico

Qx
RT
QT (segn estadio)

Bueno, sobre todo en


menores de 2 aos
Metstasis a pulmn

Pregunta 67. Neuroblastoma vs. nefroblastoma.


Pregunta 68.- R: 4
El tumor de Wilms se asocia con anomalas genitourinarias, hemihipertroa y aniridia (respuesta 2 correcta). La edad de diagnstico oscila entre
CTO Medicina C/Francisco Silvela, 106 28002 - Madrid

el ao y los 5 aos. Se han encontrado deleciones en el cromosoma 11


(respuesta 1 correcta).
La clnica ms frecuente es la masa abdominal asintomtica (respuesta 5
correcta) que no suele pasar lnea media. Otros sntomas son la HTA, en
el 60% de pacientes por compresin arterial por el tumor y produccin
de renina (respuesta 3 correcta), hematuria, etc.
La localizacin ms comn de las metstasis es el pulmn.
El diagnstico se basa en la TC y la PAAF, no aconsejndose la realizacin
de biopsia (respuesta 4 falsa).
Pregunta 69.- R: 5
Ante un nio que debuta de forma brusca con un sndrome constitucional junto con pancitopenia y hepatoesplenomegalia, el diagnstico
ms probable ser el de leucemia aguda (respuesta 5).
Pregunta 70.- R: 4
El tumor que tiene tendencia a presentar calcicaciones intraneoplsicas
(hasta en un 80% de los casos) es el neuroblastoma y no el tumor de
Wilms (respuesta 4 falsa).
Con las pruebas de imagen se puede conrmar que el tumor de Wilms
(nefroblastoma) es de origen intrarrenal (respuesta 2 correcta). En ocasiones, los pacientes con nefroblastoma pueden presentar policitemia
debido a la produccin de eritropoyetina (respuesta 1 correcta).
Las metstasis pulmonares son las ms frecuentes dentro del tumor
de Wilms, pero su presencia no suele contraindicar la ciruga del tumor
(respuesta 3 correcta).
El tumor de Wilms puede ser bilateral, sobre todo en las formas familiares
(5%), (respuesta 5 correcta).
Pregunta 71.- R: 4
Estamos ante un cuadro de sarampin. El caso clnico va relatando
detalladamente las fases de esta enfermedad:
Fase prodrmica: catarro con tos y ebre, manchas de Koplik (lesiones
blanquecinas sobre base eritematosa en mucosa subyugal), es un
dato patognomnico de esta enfermedad.
Fase exantemtica: inicio del exantema por la cara y descenso (respuesta 1 correcta). El exantema es maculopapular, no pruriginoso
(respuesta 4 falsa) y la gravedad de la enfermedad se relaciona con
la intensidad y conuencia del exantema (respuesta 3 correcta). Es
en esta fase cuando aparece ebre alta.
Fase de resolucin: el exantema desaparece en el mismo orden que
apareci.
Entre las complicaciones guran:
Cuadros ORL (otitis media aguda): complicacin ms frecuente.
Neumonas: en los nios es ms comn la sobreinfeccin bacteriana,
que la neumona de clulas gigantes de Hecht (respuesta 2 correcta).
Afectacin del SNC: encefalitis aguda y PEES (panencefalitis esclerosante subaguda): forma de encefalitis por virus lentos, con
un pronstico psimo, el diagnstico se hace detectando en LCR
aumento de Acs frente al sarampin.
Anergia cutnea con reactivacin de una tuberculosis preexistente
(respuesta 5 correcta).
Pregunta 72.- R: 3
Con los datos de febrcula y adenopatas retroauriculares y cervicales
dolorosas debemos pensar en una rubola. Se precede de un cuadro
catarral leve (1 o 2 das) con ebre baja o moderada y conjuntivitis. El
exantema es morbiliforme y conuente en cara. Se resuelve mediante
una mnima descamacin. En la analtica se puede encontrar leucopenia,
trombopenia y linfocitos atpicos.

Tfno. (0034) 91 782 43 30/33/34 E-mail: secretaria@ctomedicina.com www. ctomedicina.

13

Comentarios de Test a distancia 1. vuelta

Pediatra
El perodo de mxima transmisin abarca desde 7 das antes de aparicin del exantema hasta 7 u 8 das despus de que ste haya aparecido.
Las complicaciones son poco frecuentes en la infancia: artritis (preferentemente articulaciones de pequeo tamao), encefalitis y prpura trombopnica. La rubola suele darse en nios mayores de 6 meses, pues antes
los anticuerpos maternos actan como protectores (respuesta 3 correcta).
Una vez superada la enfermedad suele quedar inmunidad permanente.
Pregunta 73.- R: 1
El cuadro clnico descrito es el tpico de mononucleosis infecciosa. La
clnica caracterstica es astenia importante, ebre alta, adenopatas,
faringoamigdalitis con exudado blanquecino en sbana, hepatoesplenomegalia y puede cursar con exantema maculopapuloso de predominio
en tronco, a veces desencadenado por la toma de amoxicilina. En la
analtica tpicamente aparece leucocitosis con linfocitosis, presencia de
linfocitos atpicos, leve trombopenia y discreto aumento de las transaminasas, pero puede cursar con pancitopenia como es el caso. El virus
Epstein-Barr, los virus A y B de la hepatitis, el parvovirus B19, el VIH y el
citomegalovirus, son los ms asociados a pancitopenia. Para conrmar el
diagnstico habra que pedir serologa especca, aunque el tratamiento
es sintomtico, y en cuanto a la pancitopenia, actitud expectante, pues
se resuelve espontneamente en la mayora de las ocasiones.
La pancitopenia puede ser congnita o adquirida. La ms frecuente es
esta ltima, y las causas ms importantes son los frmacos, los txicos,
las infecciones, radiaciones y procesos inmunitarios. Las leucemias y la
hemoglobinuria paroxstica nocturna (HPN) son etiologas ms raras.
Pregunta 74.- R: 1
El caso que se nos presenta es una varicela. Ya sabemos que el agente
causal es el virus de la varicela-zster, de la familia de los herpes virus.
La fase prodrmica consiste en un catarro de vas altas con febrcula. Lo
caracterstico del exantema es que suele afectar a tronco y parte proximal
de extremidades, respetando las zonas distales. Las lesiones aparecen en
distintos estadios evolutivos (o lesiones en cielo estrellado); stas son
lesiones eritematosas con vesculas y otras en fase de costra. Las lesiones pican mucho. La complicacin ms frecuente es la sobreinfeccin
bacteriana de las lesiones.
Otras complicaciones son:
Neumona varicelosa: en la infancia es ms frecuente la secundaria
a sobreinfeccin bacteriana, que la producida por el propio virus.
Afectacin del SNC: lo ms habitual es en forma de cerebelitis, que
evoluciona de forma favorable.
La varicela en los nios sanos no precisa tratamiento con aciclovir. Son
indicaciones para su uso: neonatos e inmunodeprimidos.
Pregunta 75.- R: 3
La pregunta hace referencia al llamado eritema infeccioso, megaloeritema
o quinta enfermedad. En el MIR lo suelen preguntar para confundirlo
con el exantema sbito, rosola infantil o sexta enfermedad.
El eritema infeccioso tiene como agente causal el parvovirus B19 (respuesta 1 correcta). Es tpico de la edad escolar (5-15 aos) y predomina
en meses de primavera y verano. El periodo de incubacin es 1-2 semanas (respuesta 2 correcta). El cuadro se caracteriza por un periodo
prodrmico que puede cursar con febrcula, sin ebre alta (respuesta 3
falsa), seguido de la clnica exantemtica afebril que evoluciona en tres
etapas (respuesta 4 correcta):
Fase inicial con eritema en ambas mejillas (nio abofeteado).
Fase intermedia de exantema maculopapuloso en tronco, sin afectacin de palmas y plantas.
CTO Medicina C/Francisco Silvela, 106 28002 - Madrid

Tercera fase, con aclaramiento central (aspecto en encaje o reticulado). Se resuelve espontneamente entre la 1. y 4. semana, pero
en ocasiones puede recidivar cuando el nio se estresa, tiene ebre,
con el sol, etc.

Entre las complicaciones guran las artralgias-artritis y la aplasia medular


en nios susceptibles, con enfermedades hematolgicas, (respuesta 5
correcta).
Pregunta 76.- R: 2
Caso clnico de exantema sbito (rosola infantil o sexta enfermedad). Su
agente es el herpes virus tipo 6 y afecta principalmente a nios menores
de 2 aos. Se caracteriza por:
Fase febril: ebre alta, con buen estado general, que dura de 3 a 4
das.
Fase exantemtica, aparece bruscamente, tras desaparecer la ebre,
un exantema maculopapuloso poco conuente que afecta al trax,
abdomen y raz de miembros.
Son datos tpicos del hemograma la presencia de leucocitosis con
neutrolia en las primeras 24-36 horas de evolucin, por lo que, en un
primer momento, puede ser difcil establecer el diagnstico de infeccin
vrica (respuesta 2 correcta).
La complicacin ms frecuente es la crisis febril. Tambin puede causar
encefalitis.
Pregunta 77.- R: 4
Recordemos los criterios diagnsticos de la enfermedad de Kawasaki o
sd. mucocutneo-ganglionar.
A

Fiebre

Presencia de, como mnimo, cuatro de los cinco siguientes signos:


Conjuntivitis bilateral no purulenta
Alteraciones en la mucosa de la orofaringe, con inyeccin farngea;
labios secos con suras, inyectados o ambos, y lengua en fresa
Alteraciones en las zonas perifricas de las extremidades, como
edema y eritema en manos o pies, descamacin de inicio
periungueal
Exantema, de inicio en el tronco; polimorfo, no vesicular
Linfadenopata cervical unilateral

La enfermedad no se puede explicar por ninguna otra causa conocida

Diagnstico clnico A + B + C

Pregunta 77. Criterios diagnsticos de la enfermedad de Kawasaki.


Adems de estos sntomas, se puede encontrar patologa muy variada:
artritis, pericarditis, iritis, etc. Es muy llamativa la trombocitosis, especialmente en la fase subaguda de la enfermedad.
Recuerda que la etiologa es desconocida aunque se plantea que puede ser secundaria a un dao inmunitario del endotelio mediado por
superantgenos.
El tratamiento consiste en el empleo de AAS y gammaglobulina en la fase
aguda, manteniendo posteriormente el AAS en dosis antiagregantes. El
empleo de la gammaglobulina en la fase aguda previene el desarrollo
de aneurismas coronarios en la fase subaguda.
Pregunta 78.- R: 2
Este paciente presenta una prpura de Schlein-Henoch. Es la vasculitis
ms frecuente de la infancia. Afecta a pequeos vasos y est mediada
por mecanismo inmunolgico de predominio IgA.

Tfno. (0034) 91 782 43 30/33/34 E-mail: secretaria@ctomedicina.com www. ctomedicina.com

14

Comentarios de Test a distancia 1. vuelta

Pediatra
La fase prodrmica suele consistir en proceso ORL. Posteriormente puede
aparecer, en orden de frecuencia:
Afectacin cutnea: prpura palpable, en miembros inferiores, que
suele cursar en brotes.
Artritis de rodillas y tobillos.
Dolor abdominal clico, vmitos y deposiciones con sangre.
Afectacin renal: hematuria con o sin proteinuria. ste es el factor
que marca el pronstico.
Slo en menos de un 1% se produce afectacin del SNC.
El tratamiento consiste en antiinflamatorios para las molestias
articulares y en ciclos cortos de corticoides, en caso de dolor abdominal intenso. Si existe afectacin renal intensa, se emplean
inmunosupresores.
Pregunta 79.- R: 2
Para el diagnstico de mononucleosis infecciosa, aparte de la sospecha
clnica, podemos encontrar:
Leucocitosis (10.000-20.000) con ms de un 20-40% de linfocitos
atpicos (respuesta 2 falsa).
Aumento de las transaminasas (80% de casos) (respuesta 1 correcta).
Anticuerpos heterlos: son tiles en mayores de 4 aos; por debajo
de esta edad, la sensibilidad es inferiror al 20%. Puede permanecer
positivo hasta 9 meses despus, por lo que no es til para el diagnstico de infeccin activa.
Anticuerpos especcos contra el VEB (respuesta 3 correcta): en fase
aguda: la IgM ACV (frente a la cpside viral) se incrementa rpido y
se mantiene 4 semanas. La IgG ACV aparece al nal de la fase aguda
y se mantiene casi toda la vida.
EL VEB se ha asociado con distintos tumores como el linfoma Burkitt,
carcinoma nasofarngeo, sndrome de Duncan y diversos sndromes
linfoproliferativos (respuesta 4 correcta). En la fase aguda de la infeccin puede producirse rotura esplnica ante un traumatismo, por ello,
el tratamiento consiste en reposo; en algunos casos puede ser til la
administracin de corticoides (obstruccin de la va area alta, anemia
hemoltica autoinmune, convulsiones, etc.).

Pregunta 81.- R: 5
Respecto al SIDA en pediatra, hay que recordar:
La principal va de transmisin es vertical, durante el parto. El porcentaje de infectados es aproximadamente del 1%, si se aplican
medidas de actuacin oportunas. Recuerda que existe transmisin
documentada del virus mediante leche materna.
El periodo de latencia es menor que en adulto. Se subdivide en SIDA
precoz (perodo de latencia de pocos meses, clnica antes del ao de
vida, predominando las manifestaciones neurolgicas, pronstico
malo) y SIDA tardo (ms parecido al del adulto, predominando la
clnica infecciosa).
Las manifestaciones clnicas ms frecuentes en pediatra, comparndolo con los adultos son: cuadros ORL, parotiditis, neumona
intersticial linfoide y manifestaciones neurolgicas (calcicaciones
de los ganglios basales, atroa cerebral, etc.). La causa ms frecuente
de muerte es la neumona por P. carinii.
Son infecciones menos frecuentes en los nios: TBC, hepatitis B,
linfomas y sarcoma de Kaposi.
Pregunta 82.- R: 1
El diagnstico en el hijo de madre VIH positiva es difcil por la presencia de
IgG anti-VIH de la madre que pasan a travs de la placenta, de forma que
el 100% de los nios son seropositivos al nacer. Los nios no infectados
falsamente positivos pueden tardar en negativizar anticuerpos 18 meses (respuesta 1 falsa), por ello, el diagnstico va a depender de la edad:
Recuerda que es necesario realizar quimioprolaxis con AZT durante el
embarazo, el momento del parto y luego al RN. El tratamiento se basa en
la triple terapia, al igual que en los adultos, y para evaluar la evolucin,
se emplea la carga viral y el cociente CD4/CD8, recordando que la cifra
de linfocitos hay que extrapolarla segn la edad del nio.
MENORES DE 18 MESES
1. PCR-ADN + al menos en
2 determinaciones
2. Criterios SIDA

MAYORES DE 18 MESES
1. Presencia de Ac frente a VIH
(ELISA o Western-Blot)
2. Criterios del apartado anterior

Pregunta 82. Diagnstico de infeccin VIH en el nio.


Pregunta 80.- R: 4
Estamos ante un cuadro de tos ferina, infeccin por Bordetella pertussis.
La edad con ms riesgo es el periodo de lactante, la madre no le da
anticuerpos protectores (porque stos van disminuyendo a lo largo
de la vida) y el nio todava no tiene completa la vacunacin frente al
microorganismo (respuesta 1 correcta).
El periodo de incubacin oscila entre 5-10 das (respuesta 5 correcta).
La clnica consta de dos fases:
Fase catarral inicial (respuesta 2 correcta): con febrcula, rinorrea y
tos blanda. Este es el periodo de mxima contagiosidad.
Fase de tos paroxstica, consiste en accesos de tos, seguido en muchas ocasiones de vmitos y acompandose de cianosis facial. En
los nios mayores es caracterstico el gallo inspiratorio. Esta fase
puede durar 2-4 semanas (respuesta 3 correcta).

Pregunta 83.- R: 3
La triple vrica protege contra el virus del sarampin, rubola y parotiditis.
Es una vacuna de virus vivos atenuados.
Indicaciones: a los 15 meses y una dosis de recuerdo a los 4 aos, a
nios y nias.
Efectos adversos: cuadro de ebre y exantema, ms atenuados que las
infecciones frente a las que protege (son virus vivos).
Contraindicaciones:
Inmunodeprimidos: excepto en nios VIH+ (slo si existe inmunodepresin importante).
Embarazo.
La alergia al huevo ha dejado de ser una contraindicacin absoluta
(slo si existe analaxia).

La tos ferina, pese a ser infeccin bacteriana, cursa con linfocitosis


absoluta (respuesta 4 falsa) y en la radiografa de trax los datos son
inespeccos (inltrados perihiliares). El diagnstico se basa en el cultivo del moco nasal en medio de Bordet-Gengou pero la deteccin en
sangre de IgG antifactor estimulante de los linfocitos es la tcnica ms
sensible y especca.
Para el tratamiento se utilizan macrlidos, actualmente, de eleccin la
azitromicina.

Pregunta 84.- R: 4
Actualmente existen dos tipos de vacunas: polio oral (Sabin) y polio
intramuscular (Salk).
La polio Sabin es una vacuna de virus vivos atenuados que produce inmunidad local tipo IgA y vacunacin comunitaria por contacto fecal-oral.
Como efecto adverso ms importante destacan algunos casos aislados
de polio en nios sanos, lo que ha motivado en los ltimos aos que se
estn potenciando las vacunas de virus inactivados.

CTO Medicina C/Francisco Silvela, 106 28002 - Madrid

Tfno. (0034) 91 782 43 30/33/34 E-mail: secretaria@ctomedicina.com www. ctomedicina.

15

Comentarios de Test a distancia 1. vuelta

Pediatra
Est contraindicada en personas inmunodeprimidas y personas que
conviven con stos.
La vacuna tipo Salk (i.m.) est elaborada con virus inactivados, es
menos eficaz, pero ms segura. Es la que actualmente recomienda la
AEP. La pauta de administracin es a los 2-4-6-18 meses y a los 4 aos.
Pregunta 85.- R: 5
Lo bsico que tenis que recordar de la vacuna DTP es lo siguiente:
Componentes:
- Difteria: toxoide.
- Ttanos: toxoide.
- Pertussis: dos tipos: celular y acelular. Esta ltima con muchos
menos efectos adversos a nivel neurolgico. Recomendada por
la AEP. La Pertussis celular estaba contraindicada en mayores
de 7 aos, porque a partir de esa edad hay ms riesgos que
benecios.
Indicaciones: en el calendario vacunal se administra a los 2-4-6 y 18
meses y a los 4 aos se puede administrar DT o DTPa , luego cada
10 aos dT. Las primeras dosis se pueden administrar en un mismo
preparado con otras vacunas (polio inactivada, Hib, VHB), para minimizar el nmero de pinchazos.
En pacientes inmunodeprimidos se puede administrar sin riesgo.
Pregunta 86.- R: 4
La vacuna del Hib es una vacuna conjugada, muy segura y ecaz.
Protege frente a infecciones invasivas (meningitis y epiglotitis). En el
calendario vacunal se incluye a los 2-4-6 y 18 meses. Estara indicada
en nios menores de 5 aos. Presenta efectos secundarios leves y de
corta duracin. Contraindicaciones: en edades inferiores a 2 meses por
no ser inmungena.
Pregunta 87.- R: 4
La vacuna frente a la hepatitis B est realizada por ingeniera gentica.
Producen en las personas que la reciben anticuerpos frente al antgeno
de supercie del virus. En el calendario vacunal se puede administrar con
diversas pautas: (0-1-6), (0-2-6) y (2-4-6). Para prevenir la transmisin de
la hepatitis B en el neonato, en el caso de que la madre sea portadora, se
debe asociar con gammaglobulina especca al nacimiento.
Est indicada a cualquier edad si el nio no ha sido vacunado previamente
(respuesta 4 falsa). Sus contraindicaciones son muy escasas. Recordar
que el embarazo no es una contraindicacin absoluta, pero se aconseja
evitar todo tipo de vacunas siempre que sea posible.
Pregunta 88.- R: 4
Quimioprolaxis frente a los agentes ms frecuentes de meningitis.
Meningococo: a todos los contactos, independiente de la edad,
con rifampicina en dosis de 10 mg/k/dosis cada 12 horas, 4 dosis
+/- vacuna en caso del meningococo C.
Hib: nios menores de 5 aos sin vacunar y adultos que conviven
con menores de 5 aos sin vacunar (stos pueden ser portadores):
rifampicina en dosis de 20 mg/k/dosis cada 24 horas, 4 dosis +/-vacuna.
Neumococo: no existe quimioprofilaxis activa. Se puede indicar
la vacuna del neumococo de 7 serotipos para prevenir futuros
contactos.
Pregunta 89.- R: 2
En el esquema siguiente aparece esquematizada la actitud que hay que
llevar a cabo con un nio que ha tenido un contacto con un bacilfero
positivo.
CTO Medicina C/Francisco Silvela, 106 28002 - Madrid

Pregunta 89. Algoritmo de actuacin frente a contacto


con un bacilfero positivo.
Pregunta 90.- R: 3
El SMSL, muerte repentina de un nio menor de un ao cuya etiologa se
desconoce, supone la causa principal de muerte entre el mes de vida y el
ao de edad en los pases desarrollados. Este sndrome se correlaciona
con la posicin de decbito prono y lateral para dormir. Tras este factor
posicional, el tabaquismo materno es actualmente el principal factor de
riesgo del SMSL. Otros factores son la prematuridad, el antecedente de
hermano muerto por SMSL, sexo masculino, etc.
Debido a la ausencia de tratamiento, es muy importante la prevencin
con medidas como la posicin de decbito supino para dormir y la lactancia materna (que se considera el factor preventivo ms importante).
Pregunta 91.- R: 5
El ductus arterioso persistente es una cardiopata tpica de RNPT, que
consiste en la unin entre la aorta distal a la subclavia izquierda y la
arteria pulmonar. Esta cardiopata congnita tambin se asocia a la
rubola congnita y es ms frecuente en nias que en nios.
La clnica consiste en un soplo continuo, en maquinaria o de Gibson,
ms audible en 2. espacio intercostal izquierdo; los pulsos femorales
son saltones. El nio puede presentar clnica de ICC.
En la Rx de trax se aprecia pltora pulmonar con aumento de cavidades
izquierdas. El diagnstico denitivo lo obtenemos por ecocardiografa.
En los RNPT, si no existen contraindicaciones, se comienza con la administracin de indometacina, y si no se soluciona, tratamiento quirrgico.
Recuerda las contraindicaciones de la indometacina: hemorragia activa,
trombopenia, enterocolitis necrotizante e insuciencia renal. En caso de
no ser un RNPT, el tratamiento de entrada es quirrgico. Cuando interesa
mantenerlo abierto se utiliza PGE1 i.v. (respuesta 5 falsa).
Pregunta 92.- R: 4
La TGA es la cardiopata ciangena ms frecuente de inicio en el perodo neonatal (recordad que la ms comn en conjunto es la tetraloga
de Fallot, cuya clnica es ms tarda, en torno al ao de vida). La aorta
se origina en el ventrculo derecho, a la derecha y delante de la arteria
pulmonar, que se forma en el ventrculo izquierdo.
La clnica suele consistir en un RN que en las primeras horas de vida presenta taquipnea y cianosis intensa (conforme se va cerrando el ductus),
no tiene soplo y los pulsos son dbiles. En la Rx de trax se aprecia un
pedculo cardaco estrecho con pltora pulmonar. El ECG suele ser de

Tfno. (0034) 91 782 43 30/33/34 E-mail: secretaria@ctomedicina.com www. ctomedicina.com

16

Comentarios de Test a distancia 1. vuelta

Pediatra
caractersticas normales para un RN. El diagnstico se conrma mediante
la ecografa.
El tratamiento consiste en la perfusin de PGE de forma urgente para
mantener abierto el ductus, asegurando comunicacin entre ambas
circulaciones en paralelo para que sea posible la supervivencia y, cuanto
antes, realizar la correccin quirrgica. La tcnica de eleccin es el switch
arterial o Jatene, que consiste en seccionar la salida de los grandes vasos
e intercambiarlos.
Pregunta 93.- R: 2
Los malos tratos en la infancia son la segunda causa de muerte en
Espaa durante los primeros 5 aos de vida, excluyendo el periodo
neonatal. Es muy importante detectarlos precozmente pues hay casos
fatales que producen daos importantes al nio, incluso la muerte.
La actitud de los padres (despreocupacin por la gravedad de las
lesiones, justificaciones poco lgicas para las lesiones detectadas,
preocupacin por la propia situacin personal y no por la del nio,
etc.), la actitud del nio (ansiedad excesiva al hablarle y explorarle,
pasividad inadecuada para su edad, conducta agresiva, distancia
o proximidad excesivas con los padres) y la exploracin fsica, son
las herramientas que tenemos para sospechar el diagnstico. De la
exploracin, son indicadores muy sugerentes: ETS en nias prepberes, embarazo en adolescente joven, lesiones genitales o anales sin
traumatismo previo; son indicadores inespecficos: dolor o sangrado
vaginal o rectal, enuresis, encopresis, dolor abdominal, lesiones
traumticas en distintas fases de evolucin, lesiones en localizaciones que no corresponden con la actividad del nio ni su desarrollo
psicomotor, fracturas raras en nios como la del acromin. Ante la
sospecha fundada, es obligatorio dar parte a la Autoridad Judicial
y ponerse en contacto con los Servicios Sociales y de Proteccin y
Atencin al Menor, y ampliar el estudio con una serie sea completa,
un examen del fondo de ojo, una TC craneal, entre otras.
Pregunta 94.- R: 3
La CoA consiste en un estrechamiento de la luz artica. Recuerda que
es ms frecuente en nias con sndrome de Turner. El tipo ms comn
es yuxtaductal (respuesta 1 correcta).
Los nios suelen estar asintomticos (respuesta 2 correcta) y la enfermedad puede pasar desapercibida hasta la edad adulta. En la exploracin
es tpico encontrar pulsos en extremidades inferiores disminuidos y
retrasados en comparacin con los miembros superiores, lo que supone HTA en miembros superiores (respuesta 3 falsa) e incluso puede ser
mayor la TA en el miembro derecho que en el izquierdo, sobre todo si
la coartacin es proximal a la salida de la subclavia izquierda (respuesta
4 correcta). En la Rx de trax son tpicas las escotaduras costales en los
bordes inferiores de las costillas o signo de Rsler (respuesta 5 correcta)
y, a veces, el llamado signo del 3 o escotadura de la aorta en el lugar de
la coartacin. En el ECG aparecen signos de hipertroa de VI.
El tratamiento en nios es quirrgico.
Pregunta 95.- R: 2
Pregunta sobre el desarrollo puberal normal. Recuerda que la pubertad
en nias se inicia a partir de los 9 aos y en nios a partir de los 11.
El aumento del tamao testicular en el varn es el primer signo de pubertad (respuesta 2 correcta). En las nias, el primer dato a la exploracin
sugestivo de inicio de la pubertad, es la aparicin de botn mamario.
La ganancia de talla es ms acusada en nios que en nias.
Pregunta 96.- R: 3
La telarquia prematura es el desarrollo mamario prematuro en nias sin
otros caracteres sexuales secundarios, sin aceleracin de la velocidad
CTO Medicina C/Francisco Silvela, 106 28002 - Madrid

de crecimiento ni de la edad sea. La etiopatogenia no est clara, y es


probable que sea multifactorial. Es ms frecuente en los dos primeros
aos de vida, y en general tiene buen pronstico, con regresin espontnea en la mayora de los casos en 2-3 aos. Sin embargo, a veces es
persistente, pudiendo evolucionar a una pubertad precoz o adelantada,
sin que se conozcan actualmente los factores de riesgo para ello. Para
el diagnstico son tiles:
Rx de mano y mueca izquierdas (edad sea).
Ecografa plvica.
Niveles basales de estradiol.
La prueba diagnstica para excluir una pubertad precoz es el test
de GnRH (respuesta de FSH y LH ante el estmulo con anlogos de
GnRH).
Pregunta 97.- R: 4
Existen numerosos sndromes genticos y polimalformativos que se maniestan con talla baja. Sin embargo, el sndrome de Klinefelter (47xxy)
se caracteriza por presentar talla alta junto con otras alteraciones (retraso mental, alteraciones de la conducta, DM, alteraciones de la funcin
tiroidea, testes pequeos e infertilidad, etc.).
Recuerda las principales caractersticas de los otros sndromes:
Sndrome de Turner (45x0): talla baja, Pterigium colli, ausencia de
desarrollo puberal, etc. Puede asociar cardiopata y alteraciones
renales.
Sndrome de Down (95% casos, trisoma 21): hipotona, hendiduras
palpebrales, aumento de piel en zona nucal, retraso mental variable.
Puede asociar cardiopata (siendo tpico el canal AV).
Sndrome de Silver-Russell: cuadro de talla baja que se maniesta
desde la etapa fetal y aspecto pseudohidroceflico.
Sndrome de Noonan: talla baja, epicantus, hipertelorismo ocular, raz
nasal hundida, hipertelorismo mamilar. Es caracterstica la estenosis
pulmonar.
Pregunta 98.- R: 1
La causa ms frecuente de convulsiones en el RN es la encefalopata
hipoxicoisqumica. Las crisis suelen ser focales, no generalizadas,
puesto que el SNC del neonato no est completamente mielinizado.
El diagnstico se basa en el EEG. El fenobarbital es el frmaco de primera eleccin. El pronstico depender de la causa desencadenante
(respuesta 1 falsa).
Pregunta 99.- R: 3
La enuresis es la emisin involuntaria de orina en la ropa o en la cama
despus de la edad en que los nios deben tener control de esfnteres
(5 aos de edad), en ausencia de patologa orgnica. Su prevalencia es
mayor en nios que en nias (respuesta 1 falsa). Existe un claro patrn
familiar especialmente importante para la enuresis nocturna (respuesta
3 correcta).
El 90% de los casos son primarios o persistentes (el nio no ha llegado
nunca a tener un control completo de la orina) y se deben a un trastorno
madurativo. Los casos secundarios son ms frecuentes entre los 5 y los 8
aos de edad, y caractersticamente son transitorios y de buen pronstico
(respuesta 2 falsa). La enuresis diurna es ms frecuente en nias, y casi
siempre se debe al aplazamiento de la miccin (hbito retentor) hasta
rebasar la capacidad de la vejiga (respuesta 4 falsa).
El tratamiento ms correcto debe buscarse despus de una evaluacin
psicosocial y la exploracin fsica. Lo ms efectivo son las tcnicas
conductuales. La desmopresina es efectiva pero, asimismo, tiene una
alta tasa de recadas, y supone un riesgo de retencin hdrica e hiponatremia, por lo que se recomienda slo en situaciones puntuales.

Tfno. (0034) 91 782 43 30/33/34 E-mail: secretaria@ctomedicina.com www. ctomedicina.

17

Comentarios de Test a distancia 1. vuelta

Pediatra
Pregunta 100.- R: 2
El cuadro clnico hace referencia a un nio con una convulsin febril. La
clnica tpica consiste en ebre alta, crisis tnico-clnica generalizada
de breve duracin con un periodo postcrtico corto. La edad de pre-

CTO Medicina C/Francisco Silvela, 106 28002 - Madrid

sentacin es entre los 6 meses y 5 aos. Existen antecedentes familiares


hasta en un 30% de los casos. El tratamiento en la fase aguda consiste
en la administracin de diazepam intrarrectal y medidas para disminuir
la hipertermia.

Tfno. (0034) 91 782 43 30/33/34 E-mail: secretaria@ctomedicina.com www. ctomedicina.com

18

También podría gustarte